CDS(I) Exam 2016 General Knowledge Previous Year Paper

CDS(I) Exam 2016 

GENERAL KNOWLEDGE

प्रश्न 1. हृद्वाहिका रोग और अतिरक्तदाब के खतरों का सामना कर रहे व्यक्तियों के संदर्भ में निम्नलिखित में से कौन-सा एक अच्छा कोलेस्टेरॉल’ माना जाता है ? 

(a) उच्च-घनत्व लिपोप्रोटीन (HDL) 

(b) निम्न-घनत्व लिपोप्रोटीन (LDL) – 

(c) ट्राइग्लिसरॉइड 

(d) वसीय-अम्ल है 

 

प्रश्न 2. विटामिन और रोग के निम्नलिखित युग्मों में कौन सा/से सही सुमेलित है/हैं ? 

1. विटामिन A : रिकेट्स 

2. विटामिन B : बेरी-बेरी 

3. विटामिन C : स्कर्वी 

नीचे दिये गये कूट का प्रयोग कर सही उत्तर चुनिये : 

(a) केवल 2 

(b) केवल 2 और 3 

(c) केवल 1 और 3 

(d) 1, 2 और 3 

 

प्रश्न 3. किसी पिण्ड के संवेग परिवर्तन की दर किसके बराबर होती है ? 

(a) परिणामी ऊर्जा 

(b) परिणामी शक्ति 

(c) परिणामी बल 

(d) परिणामी आवेग 

 

प्रश्न 4. किसी गर्म आतप दिवस के पश्चात् लोग छतों पर जल छिड़कते हैं, क्योंकि : 

(a) जल छत के आस-पास की वायु की ऊष्मा को तुरंत अवशोषित करने में सहायक है। 

(b) जल की विशिष्ट ऊष्मा-धारिता निम्नतर होती 

(c) जल आसानी से उपलब्ध है। 

(d) जल की वाष्पीकरण की गुप्त ऊष्मा अधिक होती है। 

 

प्रश्न 5. नीचे दिये गये भारत के मानचित्र पर विचार कीजिये :

ऊपर दिये गये मानचित्र में अंकित क्षेत्र निम्नलिखित में से किस एक नकदी फसल के उत्पादन के लिए जाने जाते हैं ? 

(a) कपास 

(b) मूंगफली 

(c) गन्ना 

(d) तंबाकू 

 

प्रश्न 6. हिमालय की पीर पंजाल पर्वतमाला किसका भाग है ?

(a) शिवालिक 

(b) पार हिमालय 

(c) केन्द्रवर्ती हिमालय 

(d) निम्न हिमालय 

 

प्रश्न 7. भारत की लैटेराइट मृदाओं के संबंध में निम्नलिखित कथनों पर विचार कीजिए : 

1. लैटेराइट मृदाएं सामान्यत: लाल रंग की होती हैं 

2. लैटेराइट मृदाओं में नाइट्रोजन और पोटाश की प्रचुरता होती है। 

3. लैटेराइट मृदाएं राजस्थान और उत्तर प्रदेश में सुविकसित हैं 

4. इस मृदा में टैपियोका और काजू की उपज अच्छी होती है 

ऊपर दिये गये कथनों में से कौन सा/से सही है/हैं ? 

(a) केवल 1 

(b) 2,3 और 4 

(c) केवल 1 और 4 

(d) 1,2 और 4 

 

प्रश्न 8. भूमध्यसागरीय और मानसून जलवायु के संबंध में निम्नलिखित कथनों में से कौनसा/से सही है/ 

1. भूमध्यसागरीय जलवायु में वर्षण शीतकाल में होता है, जबकि मानसून जलवायु में यह अधिकतर ग्रीष्मकाल में होता है । 

2. भूमध्यसागरीय जलवायु में, मानसून जलवायु की अपेक्षा, तापमान का वार्षिक परिसर अधिक होता है 

3. दोनों ही जलवायुओं में वर्षा ऋतु और शुष्क ऋतु होती है नीचे दिये गये कूट का प्रयोग कर सही उत्तर चुनिए : 

(a) केवल 1 

(b) केवल 2 और 3 

(c) केवल 1 और 3 

(d) 1, 2 और 3 

 

प्रश्न 9. प्रेशर कुकरों का हैंडल प्लास्टिक का बना होता है, क्योंकि इसको ऊष्मा का कुचालक बनाया जाना चाहिए। इसमें कौनसा प्लास्टिक प्रयुक्त होता है, जो कि प्रथम मानव-निर्मित प्लास्टिक है ? 

(a) पॉलिथीन 

(b) टैरीलीन 

(c) नायलॉन 

(d) बेकेलाइट 

 

प्रश्न 10. मेथिल आइसोसायनेट गैस, जिसके कारण दिसंबर 1984 में भोपाल में आपदा हुई थी, यूनियन कार्बाइड फैक्टरी में किसके उत्पादन के लिए प्रयुक्त होती थी ? 

(a) रंजक (डाई) 

(b) अपमार्जक (डिटर्जेंट)

(c) विस्फोटक 

(d) पीड़कनाशी 

 

प्रश्न 11. पादपों और प्राणियों में प्रवर्धन के लिए जननद्रव्य (जर्मप्लाज्म) की आवश्यकता होती है। जननद्रव्य क्या होता है ? 

1. आनुवंशिक संसाधन 

2. प्रजनन के लिए बीज या तंतु 

3. अंड और शुक्राणु आधान 

4. किसी जनन कोशिका का निर्धारण जोन 

नीचे दिये गये कूट का प्रयोग कर सही उत्तर चुनिए : 

(a) केवल 1 

(b) 1, 2 और 3 

(c) केवल 2 और 3 

(d) 2 और 4 

 

प्रश्न 12. डेंगू विषाणु किसके द्वारा रोगी के रक्त में पाट्टकाणु गणना को कम करने के लिए जाना जाता है ? 

1. अस्थि मज्जा में पट्टिकाणुओं के उत्पादन की प्रक्रिया में व्यतिकरण कर 

2. एन्डोथीलियल कोशिकाओं में संक्रमण कर 

3. पट्टिकाणुओं के साथ बंधकर 

4. आंत्र में पट्टिकाणुओं का संचय कर नीचे दिये गये कूट का प्रयोग कर सही उत्तर चुनिए : 

(a) केवल 1 और 2 

(b) केवल 1 और 3 

(c) 3 और 4 

(d) 1, 2 और 3 

 

प्रश्न 13. यांत्रिक ऊर्जा की SI इकाई क्या है ? 

(a) जूल 

(b) वाट 

(c) न्यूटन-सेकण्ड 

(d) जूल-सेकण्ड 

 

प्रश्न 14. दो निकाय तापीय साम्य में कहे जाते हैं, यदि और केवल यदि : 

(a) उनके विभिन्न तापमानों पर होने पर भी उनके बीच ऊष्मा प्रवाह हो सकता हो ।

(b) उनके विभिन्न तापमानों पर होने पर भी उनके बीच ऊष्मा प्रवाह नहीं हो सकता हो 

(c) उनके बीच कोई ऊष्मा प्रवाह न हो 

(d) उनके तापमान थोड़े भिन्न हों 

 

प्रश्न 15. ‘जर्मन सिल्वर’ का उपयोग सजावटी वस्तुओं, मुद्रा धातु, आभूषणों आदि के बनाने में होता है। इसको यह नाम देने का कारण क्या है ? . 

(a) यह ताम्र की मिश्र धातु है और इसमें चाँदी एक घटक के रूप में होती है 

(b) चाँदी का उपयोग सर्वप्रथम जर्मनों ने किया 

(c) यह देखने में चाँदी जैसी होती है 

(d) यह चाँदी की मिश्रधातु है 

 

प्रश्न 16. पादपों में स्टिगमैस्टेरॉल, अर्गोस्टेरॉल, साइटोस्टेरॉल आदि जैसे विभिन्न स्टेरॉल होते हैं, जो कॉलेस्टेरॉल से अति निकट सादृश्य रखते हैं। ये पादप स्टेरॉल क्या कहलाते हैं ? 

(a) फाइटोस्टेरॉल 

(b) कैल्सिफेरॉल 

(c) अर्गोकैल्सिफेरॉल 

(d) ल्यूमिस्टेरॉल 

 

प्रश्न 17. ब्रह्मपुत्र और सतलुज नदियों पर अंतर्राष्ट्रीय सहयोग के अंतर्गत निम्नलिखित में से किस एक देश के साथ भारत ने समझौता-ज्ञापन पर हस्ताक्षर किये हैं ? 

(a) पाकिस्तान

(b) चीन 

(c) बांग्लादेश 

(d) नेपाल 

 

प्रश्न 18. भारत में निम्नलिखित रेलवे क्षेत्रों (जोन) में से किस एक का मुख्यालय, माध्य समुद्र तल से अधिकतम । ऊँचाई पर स्थित है ? 

(a) पूर्व-मध्य रेलवे 

(b) दक्षिण-पूर्व रेलवे 

(c) दक्षिण-पश्चिम रेलवे 

(d) पश्चिम-मध्य रेलवे 

 

प्रश्न 19. स्टैलैक्टाइट और स्टैलैग्माइट किसके अभिलक्षण हैं ? । 

(a) हिमनदीय स्थलाकृति 

(b) ज्वालामुखीय स्थलाकृति 

(c) कार्ट स्थलाकृति 

(d) नदीय स्थलाकृति 

 

प्रश्न 20. निम्नलिखित में से कौनसा एक, दिए गए ग्रहों को, उनके आमाप (व्यास) के बढ़ते हुए क्रम में, सही अनुक्रम है ? ” 

(a) मंगल – शुक्र – पृथ्वी – बुध — यूरेनस 

(b) बुध – मंगल – शुक्र – पृथ्वी — यूरेनस 

(c) बुध – मंगल – शुक्र — यूरेनस – पृथ्वी 

(d) शुक्र – बुध – मंगल – पृथ्वी — यूरेनस 

 

प्रश्न 21. कृत्रिम वीर्यसेचन (AI) प्रक्रिया में, निम्नलिखित में से किसको/किसको, स्त्री के गर्भाशय में प्रविष्ट कराया जाता है ? 

(a) केवल अंड 

(b) निषेचित अंड 

(c) केवल शुक्राणु 

(d) अंड और शुक्राणु 

 

प्रश्न 22. आनुवंशिकत: रूपांतरित (GM) फसलों में रूपांतरित आनुवंशिक पदार्थ किसके कारण होते हैं ? 

1. नये DNA का आगमन 

2. विद्यमान DNA का निष्कासन 

3. RNA का आगमन 

4. नए विशेषकों का आगमन 

नीचे दिये कूट का प्रयोग कर सही उत्तर चुनिए : 

(a) केवल 1 और 2 

(b) 1, 2 और 3 

(c) 3 और 4 

(d) 1, 2 और 4

 

प्रश्न 23. निम्नलिखित कथनों में से कौनसा एक सही है ? ध्वनि का वेग : 

(a) माध्यम की प्रकृति पर निर्भर नहीं करता 

(b) गैसों में अधिकतम और द्रवों में न्यूनतम होता है 

(c) ठोसों में अधिकतम और द्रवों में न्यूनतम होता है 

(d) ठोसों में अधिकतम और गैसों में न्यूनतम होता है 

 

प्रश्न 24. निम्नलिखित कथनों में से कौनसा एक सही नहीं है ? 

(a) गैसों में ध्वनि तरंगों की प्रकृति अनुदैर्घ्य होती है। 

(b) 20 Hz से कम आवृत्ति की ध्वनि तरंगें पराश्रव्य तरंगें कहलाती हैं। 

(c) उच्चतर आयामों वाली ध्वनि तरंगें अपेक्षाकृत प्रबल होती हैं। 

(d) उच्च श्रव्य आवृत्तियों वाली ध्वनि तरंगें तीक्ष्ण होती हैं। 

 

प्रश्न 25. निम्नलिखित विटामिनों में से कौनसा एक, रक्त के स्कंदन में कार्य करता है ? 

(a) विटामिन A 

(b) विटामिन B 

(c) विटामिन D 

(d) विटामिन K 

 

प्रश्न 26. विटामिन B12, की कमी प्रणाशी अरक्तता का कारक है। जंतु विटामिन B12 का संश्लेषण नहीं कर सकते। मानवों को अपना सारा विटामिन B12, अपने आहार से ही प्राप्त करना होता है। विटामिन B12 में कौन सा संकलन धातु आयन होता है ? 

(a) Mg2+ (मैग्नीशियम आयन) 

(b) Fe2+ (लोह आयन) 

(c) Co3+ (कोबाल्ट आयन) 

(d) Zn2+ (जिंक आयन) 

 

प्रश्न 27. निम्नलिखित कथनों पर विचार कीजिए : . 

1. भारत में अधिकांश कोयला और फेरस खनिज समूह विंध्य के दक्षिण में प्रायद्वीपीय में मिलते हैं 

2. प्रायद्वीपीय भारत कभी उस अधि-महाद्वीप का अंश था, जिसमें ऑस्ट्रेलिया, अंटार्कटिका, अफ्रीका और दक्षिण अमरीका सम्मिलित थे ऊपर दिए गए कथनों में कौन सा/से सही है/हैं ? 

(a) केवल 1 

(b) केवल 2 

(c) 1 और 2 दोनों 

(d) न तो 1, न ही 2 

 

प्रश्न 28. ‘महात्मा गाँधी राष्ट्रीय समुद्री उद्यान’ कहाँ अवस्थित है ? 

(a) पिरोट न द्वीप 

(b) रामेश्वरम 

(c) गंगासागर द्वीप 

(d) पोर्ट ब्लेयर 

 

प्रश्न 29. निम्नलिखित कथनों में से कौन सा/से सही है/हैं ? 

1. सामान्यतः 3 जनवरी को होने वाले रविनीच पर पृथ्वी सूर्य के निकटतम होती है 

2. सामान्यतः 4 जुलाई को होने वाले रविनीच पर पृथ्वी सूर्य से सबसे अधिक दूर होती है 

3. सामान्यतः 4 जुलाई को होने वाले सूर्योच्च पर पृथ्वी सूर्य से सबसे अधिक दूर होती है । 

4. सामान्यतः 3 जनवरी को होने वाले सूर्योच्च पर पृथ्वी सूर्य के निकटतम होती है

नीचे दिये गये कूट का प्रयोग कर सही उत्तर चुनिए : 

(a) केवल 1 

(b) 2 और 4 

(c) 1 और 3 

(d) 1 और 2 

 

प्रश्न 30. निम्नलिखित द्वीपों में से कौनसा एक, ज्वालामुखी मूल का है ? 

(a) रीयूनियन द्वीप 

(b) अंडमान और निकोबार द्वीपसमूह 

(c) लक्षद्वीप द्वीपसमूह 

(d) मालदीव 

 

प्रश्न 31. ग्लूकोस ऊर्जा का एक स्रोत है। ग्लूकोस निम्नलिखित में से किस प्रकार का अणु है ? 

(a) कार्बोहाइड्रेट 

(b) प्रोटीन 

(c) वसा 

(d) न्यूक्लीक अम्ल 

 

प्रश्न 32. कोशिका का जीवित अंश जीवद्रव्य (प्रोटोप्लाज्म) कहलाता है। यह किससे बना होता है ?

(a) केवल कोशिकाद्रव्य (साइटोप्लाज्म) 

(b) कोशिकाद्रव्य और केंद्रकद्रव्य 

(c) केवल केंद्रकद्रव्य 

(d) कोशिकाद्रव्य, केंद्रकद्रव्य और अन्य कोशिकांग 

 

प्रश्न 33. अपने किस योगदान के लिए नॉर्मन बोरलॉग ने नोबेल पुरस्कार जीता ? 

1. अत्यधिक उपज वाली फसलों का विकास । 

2. सिंचाई की आधारिक संरचनाओं का आधुनिकीकरण। 

3. संश्लेषित उर्वरकों और पीड़क नाशियों का आरंभ। 

नीचे दिये गये कूट का प्रयोग कर सही उत्तर चुनिए : 

(a) केवल 1 

(b) केवल 2 

(c) केवल 2 और 3 

(d) 1, 2 और 3 

 

प्रश्न 34. किसी निकटदृष्टिता (मायोपिया) से ग्रस्त व्यक्ति को -1.25 डाइऑप्टर पॉवर है। उसके लेंस की फोकस दूरी और स्वरूप क्या है ? 

(a) 50 cm और उत्तल लेंस 

(b) 80 cm और उत्तल लेंस 

(c) 50 cm और अवतल लेंस 

(d) 80 cm और अवतल लेंस 

 

प्रश्न 35. 100g द्रव्यमान का कोई बर्फ का टुकड़ा 0 °C पर रखा गया है। 0 °C पर पिघलने के लिए इसे कितनी ऊष्मा मात्रा आवश्यक होगी ? (बर्फ के पिघलने की गुप्त ऊष्मा का मान 333.6 J/g लीजिए): 

(a) 750.6J 

(b) 83.4 J 

(c) 33360 J 

(d) 3.336J 

 

प्रश्न 36. निम्नलिखित हॉर्मोनों में किस एक में पैप्टाइड शृंखला होती है ? 

(a) ऑक्सीटॉसिन 

(b) कॉर्टिकोट्रॉपिन 

(c) इंसुलिन 

(d) कॉर्टिसोन 

 

प्रश्न 37. निम्नलिखित में से कौनसा एक, रासायनिक परिवर्तन का उदाहरण है ? 

(a) कागज का जलना 

(b) नर्म लोहे का चुंबकित होना 

(c) जल में शक्कर का घुलना 

(d) जल से बर्फ के घनों का बनना 

 

प्रश्न 38. निम्नलिखित में से कौनसा एक, दीर्घकालीन समुद्र तल परिवर्तन का कारण है ? 

(a) वायुमंडलीय विक्षोभ 

(b) समुद्री जल घनत्व में परिवर्तन 

(c) हिमखंडों का पिघलना 

(d) बर्फ-चादरों का पिघलना 

 

प्रश्न 39. ब्रह्मपुत्र नदी की निम्नलिखित सहायक नदियों पर विचार कीजिए : 

1. लोहित 

2. तीस्ता 

3. सुबंसिरी 

4. संकोष 

उपर्युक्त नदियों को पश्चिम से पूर्व की ओर क्रमबद्ध 

(a) 2-4-3-1 

(b)  2-3-4-1

(c)  4-2-3-1

(d) 3-1-2-4

 

प्रश्न 40. निम्नलिखित में से कौनसा एक, दक्षिणी गोलार्ध में पवन का अपनी बाई ओर विक्षेपित होने का कारण है ? 

(a) उत्तरी और दक्षिणी गोलार्द्ध की जल मात्राओं में भिन्नता 

(b) ताप और दाब विभिन्नताएं 

(c) पृथ्वी का आनत अक्ष 

(d) पृथ्वी का घूर्णन 

 

प्रश्न 41. किसी चक्रवात-अक्षि पर क्या होता है ? 

(a) अप्रसामान्य उच्च ताप और निम्नतम दाब 

(b) अप्रसामान्य निम्न ताप और दाब 

(c) निर्मल आकाश और न्यूनतम ताप 

(d) घना मेघ-आच्छादन और निम्न दाब 

 

प्रश्न 42. निम्नलिखित कथनों में से कौन सा/से सही है/हैं ? 

1. अम्लीय वर्षा चूना-पत्थर से बनी इमारतों से अभिक्रिया करती है। 

2. सल्फर युक्त कोयले का दहन अम्लीय वर्षा में भागी हो सकता है 

3. प्रदूषण के नियंत्रण के लिए सुपोषण एक प्रभावी युक्ति है 

नीचे दिये गये कूट का प्रयोग कर सही उत्तर चुनिए : 

(a) केवल 1 और 2 

(b) केवल 2 और 3 

(c) केवल 1 

(d) 1, 2 और 3 

 

प्रश्न 43. दंड चुंबक के संबंध में निम्नलिखित कथनों में से कौनसा एक सही है ? 

(a) किसी दंड चुंबक के उत्तरी ध्रुव का ध्रुव-प्राबल्य उसके दक्षिणी ध्रुव के ध्रुव-प्राबल्य की अपेक्षा अधिक होता है 

(b) जब किसी दंड चुंबक को उसके अक्ष के अनुलंब काटा जाता है, तो उसके उत्तरी और दक्षिणी ध्रुव पृथक हो जाते हैं 

(c) जब किसी दंड चुंबक को उसके अक्ष के अनुलंब काटा जाता है, तो दो नए दंड चुंबक बन जाते हैं 

(d) किसी दंड चुंबक के ध्रुव परिमाण में असमान और प्रकृति में विपरीत होते हैं

 

प्रश्न 44. निम्नलिखित कथनों में से कौनसा एक सही नहीं है ? 

(a) हाइड्रोजन एक तत्व है 

(b) हाइड्रोजन सबसे हल्का तत्व है 

(c) हाइड्रोजन के कोई समस्थानिक नहीं होते 

(d) हाइड्रोजन और ऑक्सीजन एक विस्फोटक मिश्रण बनाते हैं 

 

प्रश्न 45. निम्नलिखित कथनों में से कौनसा एक सही नहीं है ? 

(a) विभिन्न तत्वों के परमाणुओं की द्रव्यमान संख्या समान हो सकती हैं 

(b) किसी तत्व के परमाणुओं की द्रव्यमान संख्या भिन्न-भिन्न हो सकती हैं 

(c) किसी तत्व के सभी परमाणुओं में प्रोटॉनों की संख्या समान होती है। 

(d) किसी तत्व के सभी परमाणुओं में न्यूट्रॉनों की संख्या सदैव समान होगी 

(a) 2-4-3-1 

(b) 2-3-4-1 

(c) 4-2-3-1 

(d) 3-1-2-4 

 

प्रश्न 46. भारत में रासायनिक उद्योग के संदर्भ में निम्नलिखित ___ कथनों में से कौनसा/से सही है/हैं ? – 

1. रासायनिक उद्योग भारत में प्राचीनतम उद्योगों में से एक है 

2. रंजक (द्रव्य) सेक्टर रासायनिक उद्योग का एक महत्वपूर्ण खंड है 

3. वस्त्र उद्योग में रंजक (द्रव्य) की सर्वाधिक खपत होती है नीचे दिये गये कूट का प्रयोग कर सही उत्तर चुनिए : 

(a) केवल 1 

(b) केवल 2 और 3 

(c) केवल 1 और 3 

(d) 1, 2 और 3 

 

प्रश्न 47. अंटार्कटिका में भारत के वैज्ञानिक और अनुसंधान मिशन के संबंध में निम्नलिखित कथनों में से कौन सा/से सही है/हैं ? 

1. अंटार्कटिका में पहला वैज्ञानिक बेस स्टेशन दक्षिण गंगोत्री था 

2. दक्षिण गंगोत्री का अब पूर्ति बेस और संक्रमण शिविर के रूप में उपयोग होता है 

3. मैत्री स्टेशन, वैज्ञानिक गतिविधियों के लिए पूरे – वर्ष मानव-संचालित रहता है। 

नीचे दिये गये कूट का प्रयोग कर सही उत्तर चुनिए : 

(a) केवल 1 

(b) केवल 2 और 3 

(c) केवल 1 और 2 

(d) 1,2 और 3 

 

प्रश्न 48. घरेलू और औद्योगिक प्रयोजनों के लिए संश्लिष्ट रबर ने प्राकृतिक रबर का स्थान ले लिया है। इसके पीछे निम्नलिखित में से कौनसा एक मुख्य कारण है ? 

(a) प्राकृतिक रबर विभिन्न उद्योगों की बढ़ती हुई — माँग की पूर्ति करने में अक्षम है। 

(b) प्राकृतिक रबर केवल उष्णकटिबंधीय देशों में ही उगाया जाता है 

(c) संश्लिष्ट रबर के लिए कच्चा माल आसानी से उपलब्ध है 

(d) प्राकृतिक रबर टिकाऊ नहीं होता 

 

प्रश्न 49. कोई व्यक्ति कंक्रीट की किसी मजबूत दीवार के पास धातु की घंटी बजाता है। वह 0.38 के पश्चात् उसकी प्रतिध्वनि सुनता है। यदि ध्वनि 340 m/s की चाल से गमन करती हो, तो उस व्यक्ति से दीवार की दूरी क्या है ? 

(a) 102 m 

(b) 11 m 

(c) 51 m 

(d) 30 m 

 

प्रश्न 50. भारत के निम्नलिखित लोकप्रिय पर्वतीय स्थानों को माध्य समुद्र तल से इसकी ऊंचाई (उच्चतम से निम्नतम) के आधार पर क्रमबद्ध कीजिये : 

1. मसूरी 

2. शिमला 

3. ऊटी 

4. दार्जिलिंग 

नीचे दिये गये कूट का प्रयोग कर सही उत्तर चुनिए : 

(a) 3-2-4-1 

(b) 3-1-4-2 

(c) 2-3-4-1 

(d) 2-4-1-3 

 

प्रश्न 51. निम्नलिखित कथनों पर विचार कीजिए : 

1. ‘हिन्द स्वराज’ में, महात्मा गाँधी ने व्यक्ति और साथ ही समाज के लिए उत्तम जीवन की संकल्पना का निरूपण किया है 

2. ‘हिन्द स्वराज’, भारत में औपनिवेशिक राज के विरुद्ध गाँधी जी के लंबे संघर्ष से प्राप्त अनभव का परिणाम था 

उपर्युक्त कथनों में से कौनसा/से सही है/हैं ? 

(a) केवल 1 

(b) केवल 2 

(c) 1 और 2 दोनों 

(d) न तो 1, न ही 2 

 

प्रश्न 52. निम्नलिखित में से कौन/से संवैधानिक निकाय 

1. राष्ट्रीय अनुसूचित जनजाति आयोग 

2. राष्ट्रीय महिला आयोग 

3. राष्ट्रीय अल्पसंख्यक आयोग 

4. राष्ट्रीय मानव अधिकार आयोग 

नीचे दिये गये कूट का प्रयोग कर सही उत्तर चुनिए : 

(a) केवल 1 

(b) केवल 1, 3 और 4 

(c) केवल 3 और 4 

(d) 1, 2, 3 और 4 

 

प्रश्न 53. अफ्रीकी-एशियाई पुनरुत्थान विषय पर बांदुंग सम्मेलन (1955) के संदर्भ में, निम्नलिखित में से कौनसा कथन सही है ? 

1. बांदुंग सम्मेलन इंडोनेशिया, म्यांमार (बर्मा), सीलोन (श्रीलंका), भारत और पाकिस्तान द्वारा आयोजित किया गया था, जिसमें विश्व की आधी से ज्यादा आबादी का प्रतिनिधित्व करने वाले 29 देशों ने प्रतिनिधि भेजे 

2. इस सम्मेलन ने पाँचों प्रायोजकों के इस असंतोष को प्रतिबिंबित किया कि एशिया को प्रभावित करने वाले निर्णयों के विषय में उनसे परामर्श करने में पश्चिमी शक्तियों की अनिच्छा है। 

3. यह सम्मेलन चीनी जनवादी गणराज्य और संयुक्त राज्य अमरीका के बीच तनाव से चिंतित था नीचे दिये गये कूट का प्रयोग कर सही उत्तर चुनिए :

(a) केवल 1 और 2 

(b) केवल 2 और 3 

(c) केवल 1 और 3 

(d) 1, 2 और 3 

 

प्रश्न 54. निम्नलिखित में से किस पुरातत्त्वज्ञ ने पूर्व-हड़प्पा संस्कृति और परिपक्व हड़प्पा संस्कृति के बीच समानताओं को सर्वप्रथम अभिज्ञात किया ? 

(a) अमलानंद घोष । 

(b) राखालदास बनर्जी 

(c) दया राम साहनी 

(d) सर जॉन मार्शल 

 

प्रश्न 55. एलोरा स्थित कैलाश नाथ मंदिर और मामल्लपुरम स्थित शोर मंदिर के बीच निम्नलिखित में से कौनसी एक समानता है ? 

(a) दोनों ही नागर वास्तुकला के उदाहरण हैं 

(b) दोनों ही ठोस चट्टानों में से उत्कीर्ण किए गए हैं 

(c) दोनों ही गुप्त-काल के मंदिर हैं 

(d) दोनों ही पल्लव राजाओं के संरक्षण में निर्मित किए गए थे 

 

प्रश्न 56. निम्नलिखित में से किसका/किंनका राजपूत चित्रकला में चित्रण नहीं किया गया है ? 

1. कृष्ण की कथाएँ 

2. राग और रागिनियाँ 

3. हम्जा के कार्य 

4. बाबर के कार्य 

नीचे दिये गये कूट का प्रयोग कर सही उत्तर चुनिए : 

(a) 1, 2 और 3 

(b) 2, 3 और 4 

(c) केवल 3 और 4 

(d) केवल 4 

 

प्रश्न 57. किस एक पंचवर्षीय योजना की उच्च प्राथमिकता, मुद्रा स्फीति को नियंत्रित करने और आर्थिक स्थिति में स्थिरता लाने की थी ? 

(a) चौथी योजना (1969-74) 

(b) पाँचवीं योजना (1974-79) 

(c) छठी योजना (1980-85) 

(d) सातवीं योजना (1985-90) 

 

प्रश्न 58. फिलिप्स वक्र के बारे में निम्नलिखित में से कौन सा/से कथन सत्य है/हैं ? 

1. यह बेरोजगारी और मुद्रास्फीति के बीच परस्पर प्रतिलोम संबंध (ट्रेड-ऑफ) दर्शाता है 

2. फिलिप्स वक्र का अधोमुखी ढलवाँ वक्र सामान्य: केवल अल्पकाल के लिए ही प्रमाणिक माना जाता है 

3. दीर्घकाल में फिलिप्स वक्र को सामान्य: बेरोजगारी की त्वरण हीन मुद्रास्फीति दर (NAIRU) पर क्षैतिज समझा जाता है 

नीचे दिये गये कूट का प्रयोग कर सही उत्तर चुनिए : 

(a) केवल 1 

(b) केवल 2 और 3 

(c) केवल 1 और 2 

(d) 1,2 और 3 

 

प्रश्न 59. निम्नलिखित में से कौनसा एक देश, यूरेशियाई आर्थिक संघ का सदस्य नहीं है ? 

(a) बेलारूस 

(b) रूस 

(c) कजाखस्तान 

(d) उज्बेकिस्तान 

 

प्रश्न 60. BRICS नेताओं ने कहाँ पर हुए शिखर सम्मेलन में एक नए विकास बैंक को स्थापित करने के लिए करार पर हस्ताक्षर किए ? . 

(a) नई दिल्ली, भारत (2012). 

(b) डर्बन, दक्षिण अफ्रीका (2013) 

(c) फोटो लीज़ा, ब्राजील (2014) 

(d) ऊफा, रूस (2015) 

 

प्रश्न 61. ‘शिशु’, ‘किशोर’ और ‘तरूण’ किसकी योजनाएं हैं ? 

(a) क्षेत्रीय ग्रामीण बैंकों की। 

(b) माइक्रो यूनिट्स डेवलपमेंट एण्ड रीफाइनेंस एजेंसी लि. (MUDRA) की। 

(c) भारतीय लघु उद्योग विकास बैंक की। 

(d) भारतीय औद्योगिक विकास बैंक की। 

 

प्रश्न 62. निम्नलिखित कथनों पर विचार कीजिए : 

1. भारत के राष्ट्रपति को, लोकसभा के अध्यक्ष को नियुक्त करने और हटाने की शक्ति होगी। 

2. अध्यक्ष को अपने कार्यालय के कृत्यों का निर्वहन अपने पूरे कार्यकाल के दौरान स्वयं करना होता है और वह स्टेशन से अपनी अनुपस्थिति या अपनी बीमारी के दौरान अपने कृत्यों को उपाध्यक्ष को प्रत्यायोजित नहीं कर सकता 

उपर्युक्त में से कौन सा/से कथन सही है/हैं ? 

(a) केवल 1 

(b) केवल 2 

(c) 1 और 2 दोनों 

(d) न तो 1, न ही 2 

 

प्रश्न 63. द्वितीय प्रशासनिक सुधार आयोग (2005) किससे संबंधित था ? 

(a) उत्तम शासन के लिए सांस्थानिक व्यवस्था में सुधार 

(b) भारतीय दंड संहिता और आपराधिक न्याय प्रणाली में सुधार 

(c) सार्वजनिक जीवन में भ्रष्टाचार को कम करने के लिए लोकपाल तंत्र का सृजन करना 

(d) शहरी शासन और प्रबंधन के नए उपाय निकालना 

 

प्रश्न 64. भारत के संविधान के अनुसार, प्रतिषेध रिट किस आदेश से संबंधित है ? 

1. न्यायिक और न्यायिककल्प प्राधिकारी के खिलाफ दिया गया आदेश 

2. अवर न्यायालय को, किसी विशिष्ट मामले में, जहाँ उसे विचार करने की अधिकारिता नहीं है. कार्यवाही करने से रोकना 

3. किसी व्यक्ति को किसी सार्वजनिक पद को, जिसके लिए वह हकदार नहीं है, धारण करने से रोकना 

नीचे दिये गये कूट का प्रयोग कर सही उत्तर चुनिए : 

(a) केवल 1 और 2 

(b) केवल 2 और 3 

(c) केवल 1 

(d) 1, 2 और 3 

 

प्रश्न 65. निम्नलिखित में से कौनसा एक, ‘प्रयाग प्रशस्ति’ में वर्णित समुद्रगुप्त का गुण नहीं था ? (a) तीक्ष्ण और परिष्कृत बुद्धि 

(b) निपुण मूर्तिकार 

(c) उत्कृष्ट संगीत प्रस्तुतियां 

(d) प्रतिभाशाली काव्य-निपुणता 

 

प्रश्न 66. कामंदक के ‘नीतिसार’ का योगदान किस विषय में है ? 

(a) तर्कशास्त्र और दर्शन 

(b) गणित 

(c) राजनीतिक नैतिकता 

(d) व्याकरण 

 

प्रश्न 67. भास्कर का ‘लीलावती’ किस विषय का मानक मूल ग्रंथ है ? 

(a) गणित 

(b) शल्यविज्ञान 

(c) काव्यशास्त्र 

(d) भाषाविज्ञान 

 

प्रश्न 68. मुख्यधारा अर्थशास्त्र में प्रयुक्त होने वाली ‘दक्षता’ की संकल्पना के बारे में निम्नलिखित में से कौनसे कथन सत्य हैं ? 

1. दक्षता तब पाई जाती है जब उत्पादन के किसी भी संभव पुनर्गठन से किसी अन्य की स्थिति बदतर किए बिना किसी की स्थिति बेहतर न की जा सके 

2. कोई अर्थव्यवस्था स्पष्ट रूप से अदक्ष है यदि यह उत्पादन संभावना फ्रंटियर (PPF) के अन्दर है 

3. न्यूनतम रूप से, दक्ष अर्थव्यवस्था इसके उत्पादन संभावना फ्रंटियर (PPF) पर होती है 

4. ‘परेटो दक्षता’, ‘परेटो इष्टतमता’ और ‘विनियोजी दक्षता’ सभी अनिवार्य रूप से एक ही चीज है जो ‘संसाधन विनिधान में दक्षता’ द्योतित करती है 

नीचे दिये गये कूट का प्रयोग कर सही उत्तर चुनिए : 

(a) केवल 1 और 4 

(b) केवल 1 और 3 . 

(c) केवल 2 और 3 

(d) 1, 2, 3 और 4 

 

प्रश्न 69. भारत-प्रशांत द्वीप सहयोग मंच (FIPIC) का प्रथम शिखर-सम्मेलन कहाँ हुआ था ? 

(a) जयपुर 

(b) सूवा 

(c) नई दिल्ली 

(d) पोर्ट मोरस्बी 

 

प्रश्न 70. अगस्त 2015 के मास में निम्नलिखित में से किस एक द्वीप राष्ट्र में संसदीय निर्वाचन हुए ? । 

(a) मालदीव

(b) फिजी 

(c) श्रीलंका 

(d) सिंगापुर 

 

प्रश्न 71. वर्ष 2015 में निम्नलिखित में से किस एक राष्ट्र ने । कठोर आर्थिक संकट का सामना किया जिसके फलस्वरूप उसके IMF के कर्ज के भुगतान में चूक हुई ? 

(a) चीन 

(b) ग्रीस 

(c) आयरलैंड 

(d) बेल्जियम 

 

प्रश्न 72. निम्नलिखित में से भारत के कौन से प्रधानमंत्री अविश्वास मत से पराजित हुए ? 

1. मोरारजी देसाई 

2. विश्वनाथ प्रताप सिंह 

3. एच.डी. देवगौड़ा 

4. अटल बिहारी वाजपेयी 

नीचे दिये गये कूट का प्रयोग कर सही उत्तर चुनिए : 

(a) 1, 2, 3 और 4 

(b) केवल 1,2 और 3 

(c) केवल 2, 3 और 4 

(d) केवल 1 और 4 

 

प्रश्न 73. राज्यसभा के बारे में निम्नलिखित में से कौन सा/से कथन सही है/हैं ? 

1. राज्यसभा की अधिकतम अनुज्ञेय संख्या 250 है 

2. राज्यसभा में, 238 सदस्य राज्यों और संघ राज्य क्षेत्रों से अप्रत्यक्ष रूप से निर्वाचित होते हैं ।

3. अखिल भारतीय सेवाओं के सर्जन जैसे मामलों में इसकी लोकसभा के साथ समान रूप से विधायी शक्तियां हैं 

नीचे दिये गये कूट का प्रयोग कर सही उत्तर चुनिए : 

(a) केवल 1 और 2 

(b) 1, 2 और 3 

(c) केवल 2 और 3 

(d) केवल 1 

 

प्रश्न 74. भारत के राष्ट्रपति के पद से संबंधित निम्नलिखित में से कौनसे कथन सही हैं ? 

1. राष्ट्रपति को विशेष मामलों में किसी अपराधी को क्षमादान की शक्ति है । 

2. राष्ट्रपति अध्यादेश प्रख्यापित तब भी कर सकता है जब संसद सत्र में हो . 

3. आपातकाल के दौरान राष्ट्रपति राज्यसभा को भंग कर सकता है 

4. राष्ट्रपति को आंग्ल-भारतीय समुदाय से दो सदस्य लोकसभा में नामनिर्देशित करने की शक्ति है 

नीचे दिये गये कूट का प्रयोग कर सही उत्तर चुनिए : 

(a) 1 और 2 

(b) केवल 1 और 4 

(c) केवल 3 और 4 

(d) 1, 3 और 4 

 

प्रश्न 75. गोरखनाथ के अनुयायियों को क्या कहा जाता था ? 

(a) जोगी

(b) नाथ-पंथी । 

(c) तांत्रिक 

(d) सन्यासी 

 

प्रश्न 76. सिख संघ के 12 राज्यों को क्या कहा जाता था ? 

(a) मिस्ल 

(b) गुरमत 

(c) सरदारी 

(d) राखी 

 

प्रश्न 77. निम्नलिखित कथनों में से कौन सा/से सही है/हैं ? – 

1. समान (कॉमन) सिविल संहिता का आदर्श भारत के संविधान के अनुच्छेद 44 में उपवर्णित किया गया है 

2. कुछ विषयों में, भारत में उच्च न्यायालयों को उच्चतम न्यायालय से अधिक विस्तृत शक्तियां दी गई है 

3. भारत का उच्चतम न्यायालय, देश के लिए – प्रथम पूर्ण रूप से स्वतंत्र न्यायालय, भारत के संविधान के अधीन 1950 में स्थापित किया गया 

नीचे दिये गये कूट का प्रयोग कर सही उत्तर चुनिए : 

(a) 1, 2 और 3 

(b) केवल 2 और 3 

(c) केवल 1 

(d) केवल 3 

 

प्रश्न 78. निम्नलिखित में से कौनसा/से अंतरण भुगतान के उदाहरण हैं ? 

1. बेरोजगारी भत्ता 

2. वेतन का भुगतान 

3. सामाजिक सुरक्षा भुगतान 

4. वृद्धावस्था पेन्शन 

नीचे दिये गये कूट का प्रयोग कर सही उत्तर चुनिए : 

(a) केवल 1 और 3 

(b) केवल 1, 2 और 3 

(c) केवल 1, 3 और 4 

(d) उपर्युक्त में से कोई नहीं 

 

प्रश्न 79. एरावन देव मंदिर, जहां अगस्त 2015 में एक बड़ा बम विस्फोट हुआ था, कहाँ पर स्थित है ?

(a) सिंगापुर 

(b) बैंकॉक 

(c) कुआलालम्पुर 

(d) काबुल 

 

प्रश्न 80. निम्नलिखित में से कौन, वह प्रथम भारतीय खिलाड़ी है, जो 2015 की विश्व बैडमिंटन चैम्पियनशिप (महिला) में फाइनल में पहुँची ? 

(a) ज्वाला गुट्टा 

(b) साइना नेहवाल 

(c) पी.वी. सिन्धु 

(d) मधुमिता बिष्ट 

 

प्रश्न 81. निम्नलिखित में से भारत के संविधान के किस एक अनुच्छेद/अनुसूची का संबंध स्वशासी जिला परिषदों से हैं-

(a) आठवीं अनुसूची 

(b) अनुच्छेद 370 

(c) छठी अनुसूची 

(d) अनुच्छेद 250  

 

प्रश्न 82. UID / आधार कार्ड के संबंध में निम्नलिखित में से कौन से कथन सही हैं ? 

1. यह भारत के सभी निवासियों के लिए पहचान का 12 अंकों का एक अनन्य रूप है 

2. यह व्यक्तियों की जैवमितीय सूचना के साथ एक पहचान संख्या है 

3. यह एक राष्ट्रीय पहचान और नागरिकता कार्ड है 

नीचे दिये गये कूट का प्रयोग कर सही उत्तर चुनिए : 

(a) केवल 2 और 3 

(b) केवल 1 और 2 

(c) केवल 1 और 3 

(d) 1, 2 और 3 

 

प्रश्न 83. भारत के नियंत्रक-महालेखापरीक्षक (CAG) के बारे में निम्नलिखित में से कौनसे कथन सही हैं ? 

1. CAG पद ग्रहण करने की तिथि से 6 वर्ष तक पद धारण करेगा। वह 65 वर्ष की आयु पूरी कर लेने पर, यदि ऐसा 6 वर्ष की अवधि समाप्त होने से पहले हो, पद रिक्त कर देगा 

2. CAG की शक्तियां भारत के संविधान से व्युत्पन्न होती हैं 

3. CAG प्रधानमंत्री और मंत्रिपरिषद् की सलाह पर भारत के राष्ट्रपति द्वारा नियुक्त एक बहु-सदस्य निकाय है 

4. CAG को, सिद्ध कदाचार या अक्षमता के आधार पर, संसद के दोनों सदनों द्वारा समावेदन किए जाने पर ही, राष्ट्रपति द्वारा हटाया जा सकता है 

नीचे दिये गये कूट का प्रयोग कर सही उत्तर चुनिए : 

(a) 1, 2 और 4 

(b) 1, 2 और 3 

(c) 3 और 4 

(d) केवल 1 और 2 

 

प्रश्न 84. विश्व स्तर पर प्रत्येक वर्ष की मानव विकास रिपोर्ट किसके द्वारा प्रकाशित की जाती है ?

(a) WTO 

(b) विश्व बैंक 

(c) UNDP 

(d) IMF 

 

प्रश्न 85. गुप्त शासकों के सिक्कों के बारे में निम्नलिखित में से कौनसा एक कथन सही है ? 

(a) मुखभाग और पृष्ठभाग दोनों पर केवल राजा का चित्र और तिथि होती थी 

(b) मुखभाग और पृष्ठभाग दोनों पर केवल किसी देवता की प्रतिमा और तिथि होती थी 

(c) मुखभाग पर सामान्यत: राजा का चित्र और पृष्ठभाग पर किसी देवता की प्रतिमा या कोई 

कथानक रूढ़ि होती थी 

(d) मुखभाग पर सामान्यत: राजा का चित्र और पृष्ठभाग पर हमेशा तिथि होती थी 

प्रश्न 86. प्रारम्भिक भारत में अग्रहार क्या था ? 

(a) ब्राह्मणों को अनुदत्त गाँव या भूमि का नाम 

(b) अगर के फूलों की माला 

(c) अधिकारियों और सैनिकों को भूमि अनुदान 

(d) वैश्य कृषकों द्वारा बसाया हुआ गाँव या भूमि 

 

प्रश्न 87. अनुसूचित जनजाति और अन्य पारम्परिक वनवासी (वन अधिकारों की मान्यता) अधिनियम, 2006 के संबंध में निम्नलिखित में से कौनसे कथन सही हैं ? 

1. यह अधिनियम, वनवासी अनुसूचित जनजातियों के, जिनका 25 अक्टूबर, 1980 के पूर्व से वन भूमि पर दखल है, वन अधिकारों को मान्यता प्रदान करता है 

2. इस अधिनियम को कार्यान्वित करने का भार राज्य/संघ राज्यक्षेत्र सरकारों के स्तर पर होता 

3. यह अधिनियम वनवासी अनुसूचित जनजातियों और अन्य पारंपरिक वनवासियों को कुछ वन अधिकारों की मान्यता देता है और उनमें निहित करता है 

नीचे दिये गये कूट का प्रयोग कर सही उत्तर चुनिए : 

(a) केवल 1 और 2 

(b) केवल 2 और 3 

(c) केवल 1 और 3 

(d) 1, 2 और 3 

 

प्रश्न 88. लिएंडर पेस ने US ओपन मिक्स्ड डबल टेनिस टाइटल (2015) किसकी साझेदारी में जीता ? 

(a) क्रिस्टीना म्लादेनोविच 

(b) फ्लेविया पेन्नेटा 

(c) मार्टिना हिंगिस 

(d) सानिया मिर्जा 

 

प्रश्न 89. निम्नलिखित में से कौन, विश्व खाद्य पुरस्कार (वर्ष 2015) का विजेता है ? 

(a) संजय राजाराम 

(b) बलदेव सिंह ढिल्लों 

(c) सर फज़ल हसन आबेद 

(d) राजेन्द्र सिंह परोडा 

 

प्रश्न 90. भारत के उच्चतम न्यायालय की सांविधानिक न्यायपीठ के निम्नलिखित में से किस एक निर्णय में मृत्युदंड अधिनिर्णीत करने के लिए विरलों में विरलतम’ सिद्धांत को पहली बार अधिकथित किया गया ? 

(a) बचन सिंह बनाम पंजाब राज्य (1980) 

(b) गोपालनचारी बनाम केरल राज्य (1980) 

(c) डा. उपेन्द्र बख्शी बनाम उत्तर प्रदेश राज्य (1983) 

(d) तुकाराम बनाम महाराष्ट्र राज्य (1979) 

 

प्रश्न 91. भारत के राष्ट्रपति के बारे में निम्नलिखित कथनों पर विचार कीजिए : 

1. राष्ट्रपति को विशिष्ट जानकारी पाने के लिए मंत्रिपरिषद को संबोधित करने का और संदेश भेजने का अधिकार है । 

2. राष्ट्रपति विधान विषयक प्रस्थापनाओं से संबंधित जानकारी मांग सकता है 

3. संघ के प्रशासन संबंधी मंत्रिपरिषद के सभी _ विनिश्चय राष्ट्रपति को संसूचित किए जाने चाहिए उपर्युक्त में से कौनसे कथन सही हैं ? 

(a) केवल 1 और 3 

(b) केवल 2 और 3 

(c) केवल 1 और 2 

(d) 1,2 और 3 

 

प्रश्न 92. निम्नलिखित में से किस एक मामले में, भारत के उच्चतम न्यायालय द्वारा, मुस्लिम स्त्री (विवाह-विच्छेद पर अधिकार संरक्षण) अधिनियम 1986 की सांविधानिक वैधता को अनुमोदित किया गया ? 

(a) मुहम्मद अहमद खान बनाम शाह बानो बेगम 

(b) दानियाल लतीफी बनाम भारत संघ 

(c) मेरी रॉय बनाम केरल राज्य 

(d) शंकरी प्रसाद बनाम भारत संघ 

 

प्रश्न 93. विजयनगर साम्राज्य के बारे में निम्नलिखित में से कौनसा/से कथन सत्य है/हैं ? 

1. राजाओं ने भगवान विरूपाक्ष की ओर से शासन करने का दावा किया 

2. शासकों ने देवताओं के साथ अपनी घनिष्ठ संलग्नता इंगित करने के लिए “हिंदू सुरत्राण” उपाधि का प्रयोग किया। 

3. सभी राजसी आदेशों पर कन्नड़, संस्कृत और तमिल में हस्ताक्षर किए जाते थे 

4. राजसी आकृति मूर्ति अब मंदिरों में प्रदर्शित की जाती थी नीचे दिये गये कूट का प्रयोग कर सही उत्तर चुनिए : 

(a) केवल 4 

(b) केवल 1 और 2 

(c) 1, 2 और 3 

(d) 1, 2 और 4

 

प्रश्न 94. मध्यकालीन भारत में ‘इक्ता’ का क्या अर्थ था ? 

(a) आध्यात्मिक प्रयोजन के लिए धार्मिक कार्मिकों _को सौंपी गई भूमि 

(b) सैन्य अधिकारियों को समनुदेशित विभिन्न प्रादेशिक इकाइयों से प्राप्त भू-राजस्व । 

(c) शैक्षिक और सांस्कृतिक क्रियाकलापों के लिए दान 

(d) जमींदार के अधिकार

 

प्रश्न 95. निम्नलिखित में से कौनसी एक पुस्तक अकबर के दरबार के चित्रों से सज्जित नहीं थी ?

(a) हम्ज़ानामा 

(b) रज्मनामा 

(c) बाबरनामा 

(d) तारीख-ए-अल्फ़ी 

 

प्रश्न 96. निम्नलिखित में से कौनसी सार्वजनिक वस्त’ नहीं है ? 

(a) बिजली 

(b) राष्ट्रीय रक्षा 

(c) प्रकाश स्तंभ 

(d) सार्वजनिक उद्यान 

 

प्रश्न 97. निम्नलिखित में से कौनसा एक, द्वितीय पंचवर्षीय योजना (1956-57 से 1960-61) के उद्देश्यों में से नहीं है ? 

(a) बुनियादी और भारी उद्योगों के विकास पर _ विशेष बल के साथ तेजी से औद्योगीकरण

(b) रोजगार के अवसरों का व्यापक विस्तार 

(c) खाद्यान्नों में आत्मनिर्भरता प्राप्त करना तथा उद्योग और निर्यात की जरूरतों को पूरा करने के लिए कृषि उत्पादन को बढ़ाना 

(d) आय और संपत्ति में असमानताओं को घटाना और आर्थिक शक्ति का अपेक्षाकृत अधिक समरूप वितरण 

 

प्रश्न 98. राष्टीय बाल नीति. 2013. कितनी आय से नीचे के प्रत्येक व्यक्ति को बालक मानती है ?

(a) 12 वर्ष 

(b) 14 वर्ष 

(c) 16 वर्ष 

(d) 18 वर्ष 

 

प्रश्न 99. निम्नलिखित में से कौनसा एक, ‘बेटी बचाओ बेटी पढ़ाओ अभियान’ का मॉनीटर किया जाने वाला लक्ष्य नहीं है ? 

(a) वर्ष 2017 तक 100 बाल लिंग अनुपात (CSR) वाले जिलों में प्रत्येक विद्यालय में बालिकाओं के लिए शौचालय उपलब्ध कराना 

(b) वर्ष 2020 तक माध्यमिक शिक्षा में 100 प्रतिशत बालिकाओं का नामांकन । 

(c) बाल यौन अपराध संरक्षण (POCSO) अधिनियम, 2012 के कार्यान्वयन द्वारा बालिकाओं के लिए संरक्षण के परिवेश का संवर्धन 

(d) निर्वाचित प्रतिनिधियों/आधारिक कार्यकर्ताओं को समुदाय के हितैषियों (कम्यूनिटी चैंपियन) के रूप में प्रशिक्षित करना ताकि वे समदायों को CSR बेहतर बनाने तथा बालिकाओं के शिक्षा के संवर्धन के लिए संघटित करें 

 

प्रश्न 100. निम्नलिखित में से कौनसा एक, ‘राष्ट्रीय उच्चतर शिक्षा अभियान (RUSA)’ का उद्देश्य नहीं है ? 

(a) निजी शिक्षा संस्थानों की समग्र गुणवत्ता को बेहतर बनाना 

(b) संबंधन, शैक्षिक और परीक्षा व्यवस्थाओं में सुधार सुनिश्चित करना 

(c) उच्चतर शिक्षा तक पहुँच में क्षेत्रीय असंतुलन को सुधारना 

(d) शोध और नवप्रवर्तनों हेतु स्वयं को समर्पित करने के लिए उच्चतर शिक्षा संस्थानों में सामर्थ्यकारी माहौल निर्मित करना 

 

प्रश्न 101. भारत में निवारक निरोध के बारे में निम्नलिखित कथनों में से कौनसा/से सही है/हैं ? 

1. निरुद्ध व्यक्ति को, भारत के संविधान के अनुच्छेद 22 के खंड (4) और (5) में उल्लिखित अधिकारों के सिवाय कोई अधिकार नहीं है । 

2. निरुद्ध व्यक्ति को निरोधादेश को इस आधार पर चुनौती देने का अधिकार है कि जब निरोधादेश पारित किया गया था, वह उससे पहले से ही जेल में था 

3. निरुद्ध व्यक्ति इस आधार पर जमानत का दावा कर सकता है कि वह मैजिस्ट्रेट के किसी आदेश के बिना ही चौबीस घंटों से अधिक कारागार में रह चुका है नीचे दिये गये कूट का प्रयोग कर सही उत्तर चुनिए : 

(a) केवल 1 और 2 

(b) केवल 2 

(c) केवल 3 

(d) 1, 2 और 3 

 

प्रश्न 102. लोकसभा का कोई सदस्य सदन का सदस्य बने रहने से निरहित नहीं हो जाता, यदि वह सदस्य : 

(a) जिस राजनीतिक दल से निर्वाचित होकर आया था/आयी थी, उससे स्वैच्छिक रूप से अपनी सदस्यता छोड़ देता/देती है 

(b) जिस राजनीतिक दल से सदन में निर्वाचित हुआ था/हुई थी, उससे निष्कासित कर दिया गया/दी गई हो 

(c) एक स्वतंत्र उम्मीदवार के रूप में निर्वाचित __ होने के पश्चात् किसी राजनीतिक दल में सम्मिलित हो जाता है 

(d) अपने राजनीतिक दल द्वारा दिए गए निदेश के विपरीत मतदान से प्रविरत रहता हो 

 

प्रश्न 103. निम्नलिखित भाषाओं में से किस एक को, भारत के संविधान की आठवीं अनुसूची में मान्यता नहीं दी गई 

(a) अंग्रेजी 

(b) संस्कृत 

(c) उर्दू 

(d) नेपाली 

 

प्रश्न 104. हज़ारा राम मंदिर किस राज्य में था ? 

(a) अवध

 (b) त्रावणकोर 

(c) विजयनगर 

(d) अहोम 

 

प्रश्न 105. दूसरी शताब्दी BC में संकल्पित लेखों के बारे में निम्नलिखित कथनों पर विचार कीजिए :

1. इसमें धार्मिक संस्थाओं को दिए गए उपहारों का अभिलेख है 

2. वे एक व्यक्ति के कार्य के सराहनीय परिणामों का अन्य व्यक्ति में अन्यारोपण होने के विचार के बारे में बताते हैं उपर्युक्त कथनों में से कौनसा/से सही है/हैं ? 

(a) केवल 1 

(b) केवल 2 

(c) 1 और 2 दोनों 

(d) न तो 1, न ही 2 

 

प्रश्न 106. निम्नलिखित कथनों पर विचार कीजिए : 

1. अभिनवगुप्त ने ‘तंत्रलोक’ नामक एक व्यापक प्रबंध लिखा जो ‘कुल’ और ‘त्रिक’ तंत्रों की शिक्षाओं को व्यवस्थित रूप से प्रस्तुत करता है 

2. आठवीं शताब्दी के आसपास गुजरात में हरिभद्र सूरि द्वारा लिखित ‘समरैच्चकहा’ तकनीकी रूप से कोई तांत्रिक कृति नहीं है परन्तु तांत्रिक । विचारों और प्रथाओं से शराबोर है 

उपर्युक्त कथनों में से कौनसा/से सही है/हैं ? 

(a) केवल 1 

(b) केवल 2 

(c) 1 और 2 दोनों 

(d) न तो 1, न ही 2 

 

प्रश्न 107. निम्नलिखित में से कौनसा/से ‘द्रव्यवत् प्रपत्र’ का/के उदाहरण है/हैं ? 

1. कोष पत्र 

2. क्रेडिट कार्ड 

3. बचत खाते तथा अल्पावधि जमाएँ 

4. खुदरा मुद्रा बाजार के म्यूचुअल फंड 

नीचे दिये गये कूट का प्रयोग कर सही उत्तर चुनिए : 

(a) केवल 1 

(b) केवल 2 

(c) 1, 2 और 3 

(d) 1,3 और 4 

 

प्रश्न 108. निम्नलिखित शब्दों में से कौनसा एक, अर्थशास्त्र में प्रतिरोधी संव्यवहार द्वारा जोखिम के परिहार की तकनीक के लिए प्रयुक्त होता है ? 

(a) बाजार पाटना 

(b) हेजिंग करना 

(c) बट्टेदारी (डिस्काउन्टिंग) करना 

(d) अवस्फीति करना 

 

प्रश्न 109. वृत्त रूपक की श्रेणी में 87वाँ अकादमी अवार्ड जीतनेवाला, ‘सिटिज़नफोर’ किसके जीवन पर आधारित है ? 

(a) अब्राहम लिंकन 

(b) अलबर्ट आइन्सटाइन 

(c) एडवर्ड स्नोडेन 

(d) लॉरा प्वात्रा 

 

प्रश्न 110. निम्नलिखित में से कौनसा एक क्षेत्र उनमें से नहीं है जिन पर 2015-16 के रेल बजट में अधिक बल दिया गया है ? 

(a) डिस्पोजेबल बिस्तरों की ऑनलाइन बुकिंग 

(b) अधिपत्रों के निरसन हेतु रक्षा यात्रा प्रणाली 

(c) यात्रियों के लिए 180 दिन पहले टिकट बुकिंग की सुविधा 

(d) जैव-शौचालय

 

प्रश्न 111. इंडिया पोस्ट की निम्नलिखित सेवाओं में से किस एक को स्थायी रूप से बंद कर दिया गया है ? 

(a) मनी-आर्डर 

(b) तार 

(c) डाक जीवन बीमा 

(d) अंतर्देशीय पत्र 

 

प्रश्न 112. ‘भारत के प्रवासी नागरिक’ की श्रेणी भारत के नागरिकता अधिनियम में संशोधन द्वारा किस वर्ष में प्रविष्ट की गई ? 

(a) 1986 

(b) 1992 

(c) 1996 

(d) 2003 

 

प्रश्न 113. शिक्षा का अधिकार, भारत के संविधान में मूल अधिकारों में किस संशोधन द्वारा जोड़ा गया ? 

(a) संविधान (86वाँ संशोधन) अधिनियम, 2002 

(b) संविधान (93वाँ संशोधन) अधिनियम, 2005 

(c) संविधान (87वाँ संशोधन) अधिनियम, 2003 

(d) संविधान (97वाँ संशोधन) अधिनियम, 2011 

 

प्रश्न 114. निम्नलिखित कथनों पर विचार कीजिए : 

1. रुम्मिनदेई के स्तंभ पर उत्कीर्ण लेख अशोक के _ ‘धम्म’ का सुस्पष्ट विवरण देते हैं। 

2. निगलिसागर शिलालेख अशोक द्वारा कोणगमन स्तूप की यात्रा किए जाने के तथ्य का अभिलेख करता है 

उपर्युक्त कथनों में से कौनसा/से सही है/हैं ?

(a) केवल 1 

(b) केवल 2 

(c) 1 और 2 दोनों 

(d) न तो 1, न ही 2 

 

प्रश्न 115. निम्नलिखित कथनों पर विचार कीजिए : 

1. असम प्रांत वर्ष 1911 में सृजित किया गया था 

2. वर्ष 1874 में, असम के अंतर्गत के 11 जिले बंगाल की लेफ्टिनेंट गवर्नरी में से पृथक कर दिए गए और एक चीफ कमिश्नर के अधीन स्वतंत्र प्रशासन के रूप में स्थापित किए गए उपर्युक्त कथनों में से कौनसा/से सही है/हैं ? 

(a) केवल 1 

(b) केवल 2 

(c) 1 और 2 दोनों 

(d) न तो 1, न ही 2 . 

 

प्रश्न 116. निम्नलिखित कथनों में से कौनसा/से सत्य है/हैं ? 

1. फाहियान का ‘गाओसेंग फाहियान झुआन’ भारत में बौद्ध स्थलों और प्रथाओं का आद्यतम आँखों देखा चीनी विवरण था 

2. यह ग्रंथ लिखते समय फाहियान सिर्फ 25 वर्ष का था 

3. फाहियान के भारत आने का प्रमुख उद्देश्य उन . ग्रंथों को प्राप्त करना और वापस ले जाना था जिनमें मठ के नियम अंतर्विष्ट थे 

नीचे दिये गये कूट का प्रयोग कर सही उत्तर चुनिए : 

(a) 1, 2 और 3. 

(b) केवल 2 

(c) केवल 1 और 3 

(d) केवल 3

 

प्रश्न 117. निम्नलिखित कथनों में से कौनसे सही हैं ? 

1. कराधान का भुगतान के सामर्थ्य का सिद्धांत यह मानता है कि लोग करों की जिस राशि का भुगतान करते हैं, वह उनके आय या संपत्ति के सापेक्ष होनी चाहिए 

2. कराधान का हित-लाभ सिद्धांत यह बताता है कि व्यक्तियों पर कर, सरकारी कार्यक्रमों से उन्हें मिलने वाले हित-लाभ के अनुपात में लगाया जाना चाहिए 

3. वर्धमान कर धनी परिवारों से कर का जितना भाग लेता है, निर्धन परिवारों से उसकी अपेक्षा अधिक भाग प्राप्त करता है 

4. अप्रत्यक्ष करों का लाभ यह है कि वे अपेक्षाकृत सस्ते होते हैं और आसानी से उगाहे जाते हैं नीचे दिये गये कूट का प्रयोग कर सही उत्तर चुनिए : 

(a) केवल 1 और 3 

(b) केवल 2 और 4 

(c) केवल 1, 2 और 4 

(d) 1, 2, 3 और 4 

 

प्रश्न 118. निम्नलिखित कथनों में से कौनसा/से असत्य है/हैं ? 

1. मजदूरी बोर्ड त्रिपक्षीय स्वरूप के होते हैं, जिनमें कामगारों, नियोक्ताओं और स्वतंत्र सदस्यों में से प्रतिनिधि होते हैं। पत्रकारों और गैर-पत्रकारों के मजदूरी बोर्डों के सिवाय अन्य सभी मजदूरी बोर्ड सांविधिक स्वरूप के होते हैं 

3. द्वितीय राष्ट्रीय श्रम आयोग ने मजदूरी बोर्डो की उपयोगिता के विरुद्ध अनुशंसा दी है 

नीचे दिये गये कूट का प्रयोग कर सही उत्तर चुनिए : 

(a) केवल 1

(b) केवल 2 

(c) केवल 1 और 2 

(d) 1, 2 और 3 

 

प्रश्न 119. भारतीय सेना में मेजर जनरल का रैक किसके समतुल्य है ? 

(a) भारतीय वायु सेना में एयर मार्शल 

(b) भारतीय नौसेना में रियर एडमिरल 

(c) भारतीय वायु सेना में एयर कमोडोर 

(d) भारतीय नौसेना में कमोडोर 

 

प्रश्न 120. वर्तमान में प्रचलित भारतीय उपग्रहों में कोई भी : 

(a) संचार उपग्रह नहीं है 

(b) संचालन उपग्रह नहीं है 

(c) पृथ्वी प्रेक्षण उपग्रह नहीं है 

(d) बृहस्पति कक्षीय उपग्रह नहीं है 

CDS(I) Exam 2016 General Knowledge Previous Year Paper

CDS(I) Exam 2016

GENERAL KNOWLEDGE

Q 1. Which one of the following is considered as ‘good cholesterol’ with reference to individuals facing the risk of cardio-vascular diseases and hypertension ? 

(a) High Density Lipoprotein (HDL) 

(b) Low Density Lipoprotein (LDL) 

(c) Triglyceride 

(d) Fatty acids 

 

Q 2. Which of the following pairs of vitamin and disease is / are correctly matched ? 

1. Vitamin A : Rickets 

2. Vitamin B, : Beriberi 

3. Vitamin C : Scurvy 

Select the correct answer using the code given below: 

(a) 2 only 

(b) 2 and 3 only 

(c) 1 and 3 only 

(d) 1, 2 and 3 

 

Q 3. The rate of change of momentum of a body is equal to the resultant : 

(a) energy 

(b) power 

(c) force 

(d) impulse

 

Q 4. After a hot sunny day, people sprinkle water on the roof-top because : 

(a) water helps air around the roof-top to absorb the heat instantly 

(b) water has lower specific heat capacity 

(c) water is easily available 

(d) water has large latent heat of vaporisation

 

Q 5. Consider the following map of India

The areas marked in the map given above account for the production of which one of the following cash crops ? 

(a) Cotton 

(b) Groundnut 

(c) Sugarcane 

(d) Tobacco 

 

Q 6. Pir Panjal Range in the Himalayas is a part of : 

(a) Shiwalik 

(b) Trans Himalaya 

(c) Central Himalaya 

(d) Lesser Himalaya 

 

Q 7. Consider the following statements regarding laterite soils of India : 

1. Laterite soils are generally red in colour 

2. Laterite soils are rich in nitrogen and potash 

3. Laterite soils are well developed in Rajasthan and UP 

4. Tapioca and cashew nuts grow well in this soil 

Which of the statements given above is / are correct? 

(a) 1 only 

(b) 2, 3 and 4 

(c) 1 and 4 only 

(d) 1, 2 and 4 

 

Q 8. Which of the following statements regarding Mediterranean and Monsoon climate is / are correct ? 

1. Precipitation in Mediterranean climate is in winter while in Monsoon climate it is mostly in summer 

2. The annual range of temperature in Mediterranean climate is higher than the Monsoon climate 

3. Rainy and dry seasons are found in both the climates 

Select the correct answer using the code given below : 

(a) 1 only 

(b) 2 and 3 only 

(c) 1 and 3 only 

(d) 1, 2 and 3 

 

Q 9. The handle of pressure cookers is made of plastic because it should be made non conductor of heat. The plastic used there is the first man-made plastic, which is : 

(a) Polythene 

(b) Terylene 

(c) Nylon 

(d) Bakelite 

 

Q 10. Methyl Isocyanate gas, which was involved in the disaster in Bhopal in December 1984, was used in the Union Carbide factory for production of: 

(a) Dyes 

(b) Detergents 

(c) Explosives 

(d) Pesticides

 

Q 11. The Germplasm is required for the propagation of plants and animals. Germplasm is the : 

1. genetic resources

2. seeds or tissues for breeding 

3. egg and sperm repository 

4. a germ cell’s determining zone 

Select the correct answer using the code given below : 

(a) 1 only 

(b) 1, 2 and 3 

(c) 2 and 3 only 

(d) 2 and 4

 

Q 12. Dengue virus is known to cause low platelet count in blood of patient by : 

1. interfering in the process of platelet production in bone marrow 

2. infecting endothelial cells 

3. binding with platelets 

4. accumulating platelets in intestine 

Select the correct answer using the code given below: 

(a) 1 and 2 only 

(b) 1 and 3 only 

(c) 3 and 4 

(d) 1, 2 and 3 

 

Q 13. The SI unit of mechanical power is : 

(a) Joule 

(b) Watt 

(c) Newton-second 

(d) Joule-second 

 

Q 14. Two systems are said to be in thermal equilibrium if and only if : 

(a) there can be a heat flow between them even if they are at different temperatures 

(b) there cannot be a heat flow between them even if they are at different temperatures (c) there is no heat flow between them 

(d) their temperatures are slightly different 

 

Q 15. ‘German silver’ is used to make decorative articles, coinage metal, ornaments etc. The name is given because : 

(a) it is an alloy of copper and contains silver as one of its components 

(b) Germans were the first to use silver 

(c) its appearance is like silver 

(d) it is an alloy of silver 

 

Q 16. Plants contain a variety of sterols like stigmasterol, ergosterol, sitosterol etc., which very closely resemble cholesterol. 

These plant sterols are referred as : 

(a) Phytosterols 

(b) Calciferols 

(c) Ergocalciferol 

(d) Lumisterols 

 

Q 17. With which one of the following countries, India has signed an MoU under the International Cooperation on Brahmaputra and Sutlej rivers ? 

(a) Pakistan 

(b) China 

(c) Bangladesh 

(d) Nepal 

 

Q 18. Headquarters of which one among the following Railway Zones in India is situated at the highest elevation from the mean sea level ? 

(a) East Central Railway 

(b) South Eastern Railway 

(c) South Western Railway 

(d) West Central Railway 

 

Q 19. Stalactites and stalagmites are features of: 

(a) glacial topography 

(b) volcanic topography 

(c) karst topography 

(d) fluvial topography 

 

Q 20. Which one of the following is the correct sequence of the given planets in increasing order of their size (diameter)? 

(a) Mars – Venus – Earth – Mercury – Uranus 

(b) Mercury – Mars – Venus – Earth – Uranus 

(c) Mercury – Mars – Venus – Uranus – Earth 

(d) Venus – Mercury – Mars – Earth – Uranus 

 

Q 21. In artificial insemination (AI) process, which of the following is / are introduced into the uterus of the female ? 

(a) Egg only 

(b) Fertilized egg 

(c) Sperm only 

(d) Egg and sperm 

 

Q 22. Genetically modified (GM) crops contain modified genetic material due to : 

1. introduction of new DNA 

2. removal of existing DNA 

3. introduction of RNA 

4. introduction of new traits Select the correct answer using the code given below: 

(a) i and 2 only 

(b) 1, 2 and 3 

(c) 3 and 4 

(d) 1, 2 and 4 

 

Q 23. Which one of the following statements is correct ? The velocity of sound :

(a) does not depend upon the nature of media 

(b) is maximum in gases and minimum in liquids 

(c) is maximum in solids and minimum in liquids 

(d) is maximum in solids and minimum in gases 

 

Q 24. Which one of the following statements is not correct? 

(a) Sound waves in gases are longitudinal in nature 

(b) Sound waves having frequency below 20 Hz are known as ultrasonic waves 

(c) Sound waves having higher amplitudes are louder 

(d) Sound waves with high audible frequencies are sharp 

 

Q 25. Which one of the following vitamins has a role in blood clotting ?

(a) Vitamin A 

(b) Vitamin B

(c) Vitamin D 

(d) Vitamin K 

 

Q 26. Vitamin B12, deficiency causes pernicious anemia. Animals cannot synthesize vitamin B12. Humans must obtain all their vitamin B12,, from their diet. The complexing metal ion in vitamin B12, is: 

(a) Mg2+ (Magnesium ion) 

(b) Fe2+ (Iron ion) 

(c) CO3+ (Cobalt ion) 

(d) Zn2+ (Zinc ion) 

 

Q 27. Consider the following statements : 

1. Most of the coal and the ferrous group of minerals in India occur in the peninsula, south of the Vindhyas 

2. The peninsular India once formed part of the super-continent which included Australia, Antarctica, Africa and South America 

Which of the statements given above is / are correct? 

(a) 1 only 

(b) 2 only 

(c) Both 1 and 2 

(d) Neither 1 nor 2 

 

Q 28. The Mahatma Gandhi National Marine Park is located in : 

(a) Pirotan Island 

(b) Rameswaram 

(c) Ganga Sagar Island 

(d) Port Blair 

 

Q 29. Which of the following statements is / are correct? 

1. The earth is nearest to the Sun at Perihelion, which generally occurs on January 3 

2. The earth is farthest away from the Sun at Perihelion, which generally occurs on July 4 

3. The earth is farthest away from the Sun at Aphelion, which generally occurs on July 4 

4. The earth is nearest to the Sun at Aphelion, which generally occurs on January 3 

Select the correct answer using the code given below : 

(a) 1 only 

(b) 2 and 4 

(c) 1 and 3 

(d) 1 and 2 

 

Q 30. Which one of the following islands is of volcanic origin ? 

(a) Reunion island 

(b) Andaman & Nicobar islands 

(c) Lakshadweep islands 

(d) Maldives 

 

Q 31. Glucose is a source of energy. Which one of the following types of molecule is Glucose ? 

(a) Carbohydrate 

(b) Protein

(c) Fat 

(d) Nucleic acid 

 

Q 32. The living content of cell is called protoplasm. It is composed of: 

(a) Cytoplasm only 

(b) Cytoplasm and nucleoplasm 

(c) Nucleoplasm only 

(d) Cytoplasm, nucleoplasm and other organelles

 

Q 33. Norman Borlaug won Nobel Peace Prize for his contributions in : 

1. development of high-yielding crops. 

2. modernization of irrigation infrastructure. 

3. introduction of synthetic fertilizers and pesticides. 

Select the correct answer using the code given below: 

(a) I only 

(b) 2 only 

(c) 2 and 3 only 

(d) 1, 2 and 3 

 

Q 34. A myopic person has a power of -1.25 Dioptre. What is the focal length and nature of his lens ? 

(a) 50 cm and convex lens 

(b) 80 cm and convex lens 

(c) 50 cm and concave lens 

(d) 80 cm and concave lens 

 

Q 35. A piece of ice, 100 g in mass is kept at 0 °C. The amount of heat it requires to melt at 0 °C is (take latent heat of melting of ice to be 333.6 J/g): 

(a) 750.6 J 

(b) 83.4 J 

(c) 33360 J 

(d) 3.336 J 

 

Q 36. Which one of the following hormones contains peptide chain?

(a) Oxytocin 

(b) Corticotropin 

(c) Insulin 

(d) Cortisone 

 

Q 37. Which one of the following is an example of chemical change? 

(a) Burning of paper 

(b) Magnetization of soft iron 

(c) Dissolution of cane sugar in water 

(d) Preparation of ice cubes from water 

 

Q 38. Which one of the following is the cause of long-term sea-level change? 

(a) Atmospheric disturbance 

(b) Change in marine water density 

(c) Melting of icebergs 

(d) Melting of ice sheets 

 

Q 39. Consider the following tributaries of river Brahmaputra : 

1. Lohit    2. Tista   3. Subansiri    4. Sankosh Arrange the above rivers from west to east: 

(a) 2-4-3-1 

(b) 2-3-4-1 

(c) 4-2-3-1 

(d) 3-1-2-4 

 

Q 40. Which one of the following is the reason due to which the wind in the southern hemisphere is deflected towards its left ? 

(a) Difference in the water masses of northern and southern hemisphere 

(b) Temperature and pressure variations 

(c) Inclined axis of the Earth 

(d) Rotation of the Earth

 

Q 41. The ‘eye’ of the cyclone has : 

(a) abnormally high temperature and lowest pressure 

(b) abnormally low temperature and pressure 

(c) clear sky and lowest temperature 

(d) dense cloud cover and low pressure 

 

Q 42. Which of the following statements is / are correct? 

1. Acid rain reacts with buildings made from limestone 

2. Burning of sulphur containing coal can contribute to acid rain 

3. Eutrophication is an effective measure to control pollution 

Select the correct answer using the code given below: 

(a) 1 and 2 only 

(b) 2 and 3 only 

(c) 1 only 

(d) 1, 2 and 3 

 

Q 43. Which one of the following statements about bar magnet is correct? 

(a) The pole strength of the north-pole of a bar magnet is larger than that of the south-pole 

(b) When a piece of bar magnet is bisected perpendicular to its axis, the north and south poles get separated 

(c) When a piece of bar magnet is bisected perpendicular to its axis, two new bar magnets are formed 

(d) The poles of a bar magnet are unequal in magnitude and opposite in nature 

 

Q 44. Which one of the following statements is not correct? 

(a) Hydrogen is an element 

(b) Hydrogen is the lightest element 

(c) Hydrogen has no isotopes 

(d) Hydrogen and oxygen form an explosive mixture 

 

Q 45. Which one of the following statements is not correct? 

(a) Atoms of different elements may have same mass numbers 

(b) Atoms of an element may have different mass numbers 

(c) All the atoms of an element have same number of protons 

(d) All the atoms of an element will always have same number of neutrons 

 

Q 46. Which of the following statements regarding chemical industry in India is / are correct? 

1. Chemical industry is one of the oldest industries in India

2. Dyestuff sector is one of the important segments of chemical industry 

3. Textile industry accounts for the largest consumption of dyestuffs 

Select the correct answer using the code given below:

(a) 1 only 

(b) 2 and 3 only 

(c) 1 and 3 only 

(d) 1, 2 and 3 

 

Q 47. Which of the following statements about India’s scientific and research mission in Antarctica is / are correct? 

1. The first scientific base station in Antarctica was Dakshin Gangotri 

2. Dakshin Gangotri is now being used as supply base and transit camp 

3. The Maitri station is manned throughout the year for scientific activities 

Select the correct answer using the code given below:

(a) 1 only 

(b) 2 and 3 only 

(c) 1 and 2 only 

(d) 1, 2 and 3 

 

Q 48. The synthetic rubber has replaced natural rubber for domestic and industrial purposes. 

Which one of the following is the main reason behind that? 

(a) Natural rubber is unable to meet the growing demand of different industries 

(b) Natural rubber is grown in tropical countries only 

(c) Raw material for synthetic rubber is easily available 

(d) Natural rubber is not durable 

 

Q 49. A person rings a metallic bell near a strong concrete wall. He hears the echo after 0.3 s. If the sound moves with a speed of 340 m/s, how far is the wall from him? 

(a) 102 m 

(b) 11 m 

(c) 51 m 

(d) 30 m 

 

Q 50. Arrange the following popular hill stations of India in terms of their height (from highest to the lowest) from the mean sea level : 

1. Mussoorie 

2. Shimla 

3. Ooty 

4. Darjeeling 

Select the correct answer using the code given below : 

(a) 3-2-4-1 

(b) 3-1-4-2 

(c) 2-3-4-1 

(d) 2-4-1-3 

 

Q 51. Consider the following statements : 

1. In Hind Swaraj, Mahatma Gandhi formulates a conception of good life for the individual as well as the society 

2. Hind Swaraj was the outcome of the experience of Gandhi’s prolonged struggle against Colonial Raj in India Which of the statements given above is / are correct? 

(a) I only 

(b) 2 only 

(c) Both 1 and 2 

(d) Neither 1 nor 2

 

Q 52. Which of the following is / are Constitutional Body / Bodies? 

1. National Commission for Scheduled Tribes 

2. National Commission for Women 

3. National Commission for Minorities 

4. National Human Rights Commission 

Select the correct answer using the code given below: 

(a) l only 

(b) 1, 3 and 4 only 

(c) 3 and 4 only 

(d) 1, 2, 3 and 4 

 

Q 53. Which of the following statements relating to the Bandung Conference on Afro-Asian Resurgence (1955) are correct? 

1. Bandung Conference was organized by Indonesia, Myanmar (Burma), Ceylon (Sri Lanka), India, and Pakistan in which 29 countries representing more than half of the world’s population sent delegates 

2. The Conference reflected the five sponsors’ dissatisfaction with what they regarded as a reluctance by the Western powers to consult with them on decisions affecting Asia 

3. The Conference was concerned over tension between the People’s Republic of China and the United States Select the correct answer using the code given below:

(a) 1 and 2 only 

(b) 2 and 3 only 

(c) 1 and 3 only 

(d) 1, 2 and 3 

 

Q 54. Who among the following archaeologists was the first to identify similarities between a pre-Harappan culture and the mature Harappan culture ?

(a) Amalananda Ghosh 

(b) Rakhaldas Banerji 

(c) Daya Ram Sahni 

(d) Sir John Marshall 

 

Q 55. Which one of the following is the common element between the Kailasanatha Temple at Ellora and the Shore Temple at Mamallapuram ? 

(a) Both are examples of Nagara architecture 

(b) Both are carved out from solid rocks 

(c) Both are Gupta period temples 

(d) Both were built under the patronage of Pallava Kings 

 

Q 56. Which of the following is / are not depicted in the Rajput paintings? 

1. The stories of Krishna 

2. Ragas and Raginis 

3. The deeds of Hamza 

4. The deeds of Babur Select the correct answer using the code given below : 

(a) 1, 2 and 3 

(b) 2, 3 and 4 

(c) 3 and 4 only 

(d) 4 only 

 

Q 57. Which one of the Five Year Plans had a high priority to bring inflation under control and to achieve stability in the economic situation ? 

(a) Fourth Plan (1969-74) 

(b) Fifth Plan (1974-79) 

(c) Sixth Plan (1980-85) 

(d) Seventh Plan (1985-90) 

 

Q 58. Which of the following statement(s) is/are true with respect to Phillips Curve ? 

1. It shows the trade-off between unemployment and inflation 

2. The downward sloping curve of Phillips Curve is generally held to be valid only in the short run 

3. In the long run, Phillips Curve is usually thought to be horizontal at the non-accelerating inflation rate of unemployment (NAIRU) 

Select the correct answer using the code given below: 

(a) 1 only 

(b) 2 and 3 only 

(c) 1 and 2 only 

(d) 1, 2 and 3 

 

Q 59. Which one of the following nations is not a member of the Eurasian Economic Union ? 

(a) Belarus 

(b) Russia 

(c) Kazakhstan 

(d) Uzbekistan

 

Q 60. BRICS leaders signed the agreement to establish a New Development Bank at the summit held in : 

(a) New Delhi, India (2012) 

(b) Durban, South Africa (2013) 

(c) Fortaleza, Brazil (2014) 

(d) Ufa, Russia (2015) 

 

Q 61. Shishu, Kishor and Tarun are the schemes bf: 

(a) Regional Rural Banks. 

(b) Micro Units Development & Refinance Agency Ltd. (MUDRA). 

(c) Small Industries Development Bank of India. 

(d) Industrial Development Bank of India.

 

Q 62. Consider the following statements : 

1. The President of India shall have the power to appoint and remove the Speaker of Lok Sabha 

2. The Speaker has to discharge the functions of his office himself throughout his term and cannot delegate his functions to the Deputy Speaker during his absence from the station or during his illness 

Which of the statements given above is / are correct? 

(a) 1 only 

(b) 2 only 

(c) Both 1 and 2 

(d) Neither 1 nor 2 

 

Q 63. The Second Administrative Reforms Commission (2005) was concerned with : 

(a) reforms in institutional arrangements for good governance 

(b) reforms in the Indian Penal Code and the Criminal Justice System 

(c) creating an ombudsman mechanism for reduction of corruption in public life 

(d) devising new measures for urban governance and management 

 

Q 64. As per the Constitution of India, the Writ of Prohibition relates to an order : 

1. issued against judicial and quasi judicial authority 

2. to prohibit an inferior Court from proceeding in a particular case where it has no jurisdiction to try 

3. to restrain a person from holding a public office to which he is not entitled 

Select the correct answer using the code given below: 

(a) 1 and 2 only 

(b) 2 and 3 only 

(c) I only 

(d) 1, 2 and 3 

 

Q 65. Which one among the following was not an attribute of Samudragupta described in Prayag Prashasti? 

(a) Sharp and polished intellect 

(b) Accomplished sculptor 

(c) Fine musical performances 

(d) Poetical talent of a genius 

 

Q 66. Kamandaka’s Nitisara is a contribution to : 

(a) Logic and Philosophy 

(b) Mathematics

(c) Political morality 

(d) Grammar 

 

Q 67. The Lilavati of Bhaskara is a standard text on: 

(a) Mathematics 

(b) Surgery 

(c) Poetics 

(d) Linguistics 

 

Q 68. Which of the following statement(s) are true with respect to the concept of ‘EFFICIENCY’ as used in mainstream economics? 

1. Efficiency occurs when no possible reorganisation of production can make anyone better off without making someone else worse off 

2. An economy is clearly inefficient if it is inside the Production Possibility Frontier (PPF) 

3. At a minimum, an efficient economy is on its Production Possibility Frontier (PPF) 

4. The terms such as ‘Pareto Efficiency’, Pareto Optimality’ and ‘Allocative Efficiency’ are all essentially one and the same which denote ‘efficiency in resource allocation’ 

Select the correct answer using the code given below: 

(a) 1 and 4 only 

(b) 1 and 3 only 

(c) 2 and 3 only 

(d) 1, 2, 3 and 4 

 

Q 69. The first summit of the Forum for India Pacific Islands Cooperation (FIPIC) was held in : 

(a) Jaipur 

(b) Suva 

(c) New Delhi 

(d) Port Moresby

 

Q 70. Which one of the following island nations went for Parliamentary elections in the month of August 2015 ? 

(a) Maldives 

(b) Fiji 

(c) Sri Lanka 

(d) Singapore 

 

Q 71. Which one of the following nations has faced a severe economic crisis in the year 2015 resulting in default in repayment of IMF loan? 

(a) China 

(b) Greece 

(c) Ireland 

(d) Belgium 

 

Q 72. Who among the following Prime Ministers of India were defeated by a vote of No Confidence ? 

1. Morarji Desai 

2. Viswanath Pratap Singh

3. H.D. Deve Gowda 

4. Atal Bihari Vajpayee 

Select the correct answer using the code given below: 

(a) 1, 2, 3 and 4 

(b) 1, 2 and 3 only 

(c) 2, 3 and 4 only 

(d) 1 and 4 only 

 

Q 73. Which of the following statements regarding Rajya Sabha is / are correct? 

1. The maximum permissible strength of Rajya Sabha is 250 

2. In Rajya Sabha, 238 members are elected indirectly from the States and Union Territories 

3. It shares legislative powers equally with Lok Sabha in matters such as creation of All India Services 

Select the correct answer using the code given below: 

(a) 1 and 2 only 

(b) 1, 2 and 3 

(c) 2 and 3 only 

(d) 1 only 

 

Q 74. Which of the following statements relating to the office of the President of India are correct? 

1. The President has the power to grant pardon to a criminal in special cases 

2. The President can promulgate ordinances even when the Parliament is in session 

3. The President can dissolve the Rajya Sabha during emergency 

4. The President has the power to nominate two members in the Lok Sabha from the Anglo Indian community

Select the correct answer using the code given below: 

(a) 1 and 2 

(b) 1 and 4 only 

(c) 3 and 4 only 

(d) 1, 3 and 4 

 

Q 75. The followers of Gorakhnath were called : 

(a) Jogis 

(b) Nath-Panthis 

(c) Tantriks 

(d) Sanyasis 

 

Q 76. What were the 12 states of the Sikh confederacy called ? 

(a) Misl 

(b) Gurmata 

(c) Sardari 

(d) Rakhi 

 

Q 77. Which of the statements given below is / are correct ? 

1. The ideal of a common civil code is set forth in Article 44 of the Constitution of India 

2. In certain respects, the High Courts in India have been given more extensive powers than the Supreme Court 

3. The Supreme Court of India, the first fully independent Court for the country, was set up under the Constitution of India in 1950 

Select the correct answer using the code given below: 

(a) 1, 2 and 3 

(b) 2 and 3 only 

(c) l only 

(d) 3 only 

 

Q 78. Which of the following is / are the example(s) of Transfer Payment(s)? 

1. Unemployment Allowance 

2. Payment of Salary 

3. Social Security Payments 

4. Old age Pension 

Select the correct answer using the code given below: 

(a) 1 and 3 only 

(b) 1, 2 and 3 only 

(c) 1, 3 and 4 only 

(d) None of the above 

 

Q 79. The Erawan shrine, which witnessed a major bomb blast in August 2015, is located at : 

(a) Singapore 

(b) Bangkok 

(c) Kuala Lumpur 

(d) Kabul 

 

Q 80. Who among the following is the first Indian sportsperson to reach the finals in the World Badminton Championship (Women) in 2015? 

(a) Jwala Gutta 

(b) Saina Nehwal 

(c) P.V. Sindhu 

(d) Madhumita Bisht 

 

Q 81. Which one of the following Articles / Schedules in the Constitution of India deals with Autonomous District Councils ? 

(a) Eighth Schedule 

(b) Article 370 

(c) Sixth Schedule 

(d) Article 250 

 

Q 82. Which of the following statements with regard to UID / Aadhar Card are correct? 

1. It is a 12 digit unique form of identification for all residents of India 

2. It is an identity number along with the biometric information of the individuals 

3. It is a national identity and citizenship card 

Select the correct answer using the code given below:

(a) 2 and 3 only 

(b) 1 and 2 only 

(c) 1 and 3 only 

(d) 1, 2 and 3 

 

Q 83. Which of the following statements about the Comptroller and Auditor General of India (CAG) are correct? 

1. The CAG will hold office for a period of six years from the date he assumes the office. He shall vacate office on attaining the age of 65 years, if earlier than the expiry of the 6 year term 

2. The powers of CAG are derived from the Constitution of India 

3. The CAG is a multi-member body appointed by the President of India in consultation with the Prime Minister and the Council of Ministers 

4. The CAG may be removed by the President only on an address from both Houses of Parliament, on the grounds of proved misbehavior or incapacity Select the correct answer using the code given below : 

(a) 1, 2 and 4 

(b) 1, 2 and 3 

(c) 3 and 4 

(d) 1 and 2 only 

 

Q 84. Human Development Report for each year at global level is published by : 

(a) WTO 

(b) World Bank 

(c) UNDP 

(d) IMF 

 

Q 85. Which one among the following statements about the coins of the Gupta rulers is correct? 

(a) The obverse and reverse, both, had only the king’s portrait and date 

(b) The obverse and reverse, both, had only an image of a deity and date 

(c) The obverse generally had king’s portrait and reverse had an image of a deity or a motif 

(d) The obverse generally had king’s portrait and reverse always had a date 

 

Q 86. The agrahara in early India was: 

(a) the name of a village or land granted to Brahmins 

(b) the garland of flowers of Agar 

(c) the grant of land to officers and soldiers 

(d) land or village settled by Vaishya farmers

 

Q 87. Which of the following statements relating to the Scheduled Tribes and Other Traditional Forest Dwellers (Recognition of Forest Rights) Act, 2006 are correct? 

1. The Act recognizes forest rights of forest dwelling Scheduled Tribes who have been occupying the forest land before October 25, 1980 

2. The onus of implementation of the Act lies at the level of the State / UT Governments 

3. The Act seeks to recognize and vest certain forest rights in the forest dwelling Scheduled Tribes and other traditional forest dwellers Select the correct answer using the code given below: 

(a) 1 and 2 only 

(b) 2 and 3 only 

(c) 1 and 3 only 

(d) 1, 2 and 3

 

Q 88. Leander Paes won the US Open Mixed Doubles Tennis Title (2015) partnering with : 

(a) Kristina Mladenovic 

(b) Flavia Pennetta 

(c) Martina Hingis 

(d) Sania Mirza 

 

Q 89. Who among the following is the winner of the World Food Prize (year 2015)? 

(a) Sanjaya Rajaram 

(b) Baldev Singh Dhillon 

(c) Sir Fazle Hasan Abed 

(d) Rajendra Singh Paroda 

 

Q 90. In which one of the following judgments of the Constitutional Bench of the Supreme Court of India, the ‘rarest of rare principle in the award of death penalty was first laid down? 

(a) Bachan Singh v. State of Punjab (1980) 

(b) Gopalanachari v. State of Kerala (1980) 

(c) Dr. Upendra Baxi v. State of UP (1983) 

(d) Tukaram v. State of Maharashtra (1979) 

 

Q 91. Consider the following statements about the President of India: 

1. The President has the right to address and send message to the Council of Ministers to elicit specific information 

2. The President can call for information relating to proposals for legislation 

3. All decisions of the Council of Ministers relating to administration of the Union must be communicated to the President Which of the statements given above are correct? 

(a) 1 and 3 only 

(b) 2 and 3 only 

(c) 1 and 2 only 

(d) 1, 2 and 3

 

Q 92. In which one of the following cases, the Constitutional validity of the Muslim Women (Protection of Rights of Divorce) Act 1986, was upheld by the Supreme Court of India ? 

(a) Muhammad Ahmed Khan v. Shah Bano Begum 

(b) Danial Latifi v. Union of India 

(c) Mary Roy v. State of Kerala 

(d) Shankari Prasad v. Union of India 

 

Q 93. Which of the following statements about the Vijayanagara Empire is / are true? 

1. The kings claimed to rule on behalf of the God Virupaksha 

2. Rulers used the title “Hindu Suratrana” to indicate their close links with Gods 

3. All royal orders were signed in Kannada, Sanskrit and Tamil 

4. Royal portrait sculpture was now displayed in temples Select the correct answer using the code given below: 

(a) 4 only 

(b) 1 and 2 only 

(c) 1, 2 and 3 

(d) 1, 2 and 4

 

Q 94. Iqta in medieval India meant: 

(a) land assigned to religious personnel for spiritual purposes 

(b) land revenue from different territorial units assigned to army officers 

(c) charity for educational and cultural activities 

(d) the rights of the zamindar 

 

Q 95. Which one of the following books was not illustrated with paintings in Akbar’s court? 

(a) Hamzanama 

(b) Razmnama 

(c) Baburnama 

(d) Tarikh-i-Alfi 

 

Q 96. Which of the following is not a ‘Public Good’? 

(a) Electricity 

(b) National Defence 

(c) Light House 

(d) Public Parks 

 

Q 97. Which one of the following is not among the aims of the Second Five Year Plan (1956-57 to 1960-61) ? 

(a) Rapid industrialization with particular emphasis on the development of basic and heavy industries

(b) Large expansion of employment opportunities 

(c) Achieve self-sufficiency in food grains and increase agricultural production to meet the requirements of industry and exports 

(d) Reduction of inequalities in income and wealth and a more even distribution of economic power 

 

Q 98. The National Policy for Children, 2013 recognizes every person as a child below the age of: 

(a) 12 years 

(b) 14 years 

(c) 16 years 

(d) 18 years 

 

Q 99. Which one of the following is not a monitorable target of the Beti Bachao Beti Padhao Abhiyan? 

(a) Provide girls’ toilet in every school in 100 Child Sex Ratio (CSR) districts by the year 2017 ) 

(b) 100 percent girls’ enrolment in secondary education by the year 2020 

(c) Promote a protective environment for girl children through implementation of Protection of Children from Sexual Offences (POCSO)Act, 2012 

(d) Train Elected Representatives / Grassroot functionaries as Community Champions to mobilize communities to improve CSR and promote girl’s education 

 

Q 100. Which one of the following is not an objective of the Rashtriya Uchchatar Shiksha Abhiyan (RUSA)? 

(a) Improve the overall quality of private educational institutions 

(b) Ensure reforms in the affiliation, academic and examination systems 

(c) Correct regional imbalances in access to higher education 

(d) Create an enabling atmosphere in the higher educational institutions to devote themselves to research and innovations

 

Q 101. Which of the following statements with regard to preventive detention in India is / are correct ? 

1. The detenue has no rights other than those mentioned in clauses (4) and (5) of Article 22 of Constitution of India 

2. The detenue has a right to challenge the detention order on the ground that he was already in jail when the detention order was passed 

3. The detenue can claim bail on the ground that he has been in prison beyond twenty-four hours without an order of the magistrate 

Select the correct answer using the code given below: 

(a) 1 and 2 only 

(b) 2 only 

(c) 3 only 

(d) 1, 2 and 3 

 

Q 102. A Member of Lok Sabha does not become disqualified to continue as a Member of the House if the Member : 

(a) voluntarily gives up his / her membership of the political party from which he/she was elected 

(b) is expelled by the political party from which he / she had been elected to the House

(c) joins a political party after being elected as an independent candidate 

(d) abstains from voting contrary to the direction by his/her political party 

 

Q 103. Which one of the following languages is not recognized in the Eighth Schedule to the Constitution of India ? 

(a) English 

(b) Sanskrit 

(c) Urdu 

(d) Nepali 

 

Q 104. Which kingdom did the temple of Hazara Rama belong to ? 

(a) Avadh 

(b) Travancore 

(c) Vijayanagara 

(d) Ahom 

 

Q 105. Consider the following statements about votive inscriptions in the second century BC: 

1. They record gifts made to religious institutions 

2. They tell us about the idea of transference of the meritorious results of the action of one person to another person 

Which of the statements given above is / are correct? 

(a) 1 only 

(b) 2 only 

(c) Both 1 and 2 

(d) Neither 1 nor 2 

 

Q 106. Consider the following statements : 

1. Abhinavagupta wrote a comprehensive treatise called the Tantraloka which systematically presents the teachings of the Kula and Trika systems 

2. The Samaraichchakaha by Haribhadra Suri written in Gujarat around the eighth century is technically not a tantric work but is saturated with tantric ideas and practices Which of the statements given above is / are correct ? 

(a) 1 only 

(b) 2 only 

(c) Both 1 and 2 

(d) Neither 1 nor 2 

 

Q 107. Which of the following is/are example(s) of ‘Near Money’ ? 

1. Treasury Bill 

2. Credit Card 

3. Savings accounts and small time deposits 

4. Retail money market mutual funds Select the correct answer using the code given below : 

(a) 1 only 

(b) 2 only 

(c) 1, 2 and 3 

(d) 1, 3 and 4 

 

Q 108. Which one of the following terms is used in Economics to denote a technique for avoiding a risk by making a counteracting transaction ? 

(a) Dumping 

(b) Hedging 

(c) Discounting 

(d) Deflating  

 

Q 109. ‘Citizenfour’, the 87th Academy Award winner in the category of documentary feature, is based on the life of: 

(a) Abraham Lincoln 

(b) Albert Einstein 

(c) Edward Snowden 

(d) Laura Poitras 

 

Q 110. Which one of the following is not a thrust area in the Railway Budget 2015-16 ?

(a) Online booking of disposable bed rolls 

(b) Defence Travel System to eliminate Warrants 

(c) 180 days in advance ticket booking facility for passengers 

(d) Bio-toilets 

 

Q 111. Which one of the following services of India Post has permanently been discontinued ? 

(a) Money Order 

(b) Telegram 

(c) Postal Life Insurance 

(d) Inland Letter 

 

Q 112. The category of ‘Overseas Citizens of India’ was entered in the Citizenship Act of India through an amendment in the year : 

(a) 1986 

(b) 1992 

(c) 1996 

(d) 2003 

 

Q 113. The Right to Education was added to the Fundamental Rights in the Constitution of India through the : 

(a) Constitution (86th Amendment) Act, 2002 

(b) Constitution (93rd Amendment) Act, 2005 

(c) Constitution (87th Amendment) Act, 2003 

(d) Constitution (97th Amendment) Act, 2011 

 

Q 114. Consider the following statements : 

1. The inscriptions on the pillar at Rummindei give vivid details of Ashoka’s Dhamma 2. The Nigalisagar inscription records the fact of Ashoka having visited the Konagamana stupa 

Which of the statements given above is / are correct ? 

(a) 1 only 

(b) 2 only 

(c) Both 1 and 2 

(d) Neither 1 nor 2 

 

Q 115. Consider the following statements : 

1. The province of Assam was created in the year 1911 

2. Eleven districts comprising Assam were separated from the Lieutenant Governorship of Bengal and established as an independent administration under a Chief Commissioner in the year 1874 Which of the statements given above is / are correct? 

(a) l only 

(b) 2 only 

(c) Both 1 and 2 

(d) Neither 1 nor 2 

 

Q 116. Which of the following statements is / are true ? 

1. Faxian’s Gaoseng Faxian Zhuan was the earliest first-hand Chinese account of Buddhist sites and practices in India 

2. Faxian was only 25 years old at the time of writing the text 

3. Faxian’s main aim in coming to India was to obtain and take back texts containing monastic rules 

Select the correct answer using the code given below: 

(a) 1, 2 and 3 

(b) 2 only 

(c) 1 and 3 only 

(d) 3 only 

 

Q 117. Which of the following statements are correct? 

1. Ability to pay principle of taxation holds that the amount of taxes people pay should relate to their income or wealth 

2. The Benefit Principle of taxation states that individuals should be taxed in proportion to the benefit they receive from Government programmes 

3. A progressive tax takes a larger share of tax from poor families than it does from rich families 

4. Indirect taxes have the advantage of being cheaper and easier to collect Select the correct answer using the code given below : 

(a) 1 and 3 only 

(b) 2 and 4 only 

(c) 1, 2 and 4 only 

(d) 1, 2, 3 and 4 

 

Q 118. Which of the following statement(s) is /are false? 

1. Wage Boards are tripartite in nature, with representatives from workers, employers and independent members 

2. Except for the Wage Boards for Journalists and Non-Journalists, all the other wage boards are statutory in nature 

3. Second National Commission on Labour has recommended against the utility of wage boards 

Select the correct answer using the code given below: 

(a) I only 

(b) 2 only 

(c) 1 and 2 only 

(d) 1, 2 and 3 

 

Q 119. The rank of Major General in Indian Army is equivalent to : 

(a) Air Marshal in Indian Air Force 

(b) Rear Admiral in Indian Navy 

(c) Air Commodore in Indian Air Force 

(d) Commodore in Indian Navy 

 

Q 120. Among the currently operational Indian Satellites, there is no 

(a) Communication satellite 

(b) Navigation satellite 

(c) Earth observation satellite 

(d) Jupiter orbiter satellite 

Latest Current Affairs 17 May 2021

CURRENT AFFAIRS
17 May 2021

NATIONAL NEWS:

 

A) Centre admits COVID spread to rural areas, issues SOPs.

COVID’s ingress is now being seen in peri-urban, rural and tribal areas as well, the Health Ministry admitted on Sunday, weeks after a rising number of cases have been reported from rural areas of Uttar Pradesh, Bihar and Gujarat among other States. The Ministry in its document SOP on COVID-19 Containment and Management in Peri-urban, Rural and Tribal areas said there is a need to enable communities, strengthen primary level healthcare infrastructure at all levels to intensify COVID-19 response in these new areas, while continuing to provide other essential health services. The Ministry said with the larger spread of COVID-19 cases, it is important to ensure that these areas are equipped and oriented to manage COVID-19 cases. In every village, active surveillance should be done for influenza-like illness/ severe acute respiratory infections(ILI/SARI) periodically by health workers, noted the Ministry in its latest SOP. The Ministry said that depending upon the intensity of surge and number of cases, as far as feasible, contact tracing should be done as per Integrated Disease Surveillance Programme’s (IDSP’s) guidelines for contact tracing of COVID-19 cases in community settings amid reports of several bodies of suspected COVID-19 victims having been found abandoned and floating in the Ganga. The Ministry has also directed that staff should be trained in performing Rapid Antigen Testing (RAT) and that provision for RAT kits should be made at all public health facilities including sub-centres (SCs)/ Health and Wellness Centres (HWCs) and Primary Health Centres (PHCs). These patients should also be counselled to isolate themselves till test results are available. Those asymptomatic but having history of high-risk exposure to COVID patients (exposure of more than 15 mins without a mask within 6 feet distance) should be advised quarantine and tested as per ICMR protocol, notes the Ministry. The document further notes that on discharge, patients should be counselled for post-COVID management at home and leaflets regarding danger signs (e.g. breathlessness, chest pain, recurrence of fever, low oxygen saturation, etc.), precautions and various respiratory exercises. Patients with other co-morbidities should also be followed up and primary assessment of other co-morbidity (e.g. measuring blood pressure, blood glucose level) should be arranged and any modification treatment, if necessary, should be decided by a primary health centre medical officer, says the document. The Ministry has advocated for use of telemedicine services for providing post-COVID follow-up care.

B) Second batch of Sputnik V arrives.

India on Sunday received the second consignment of Sputnik V, the Russian vaccine that recently joined the country’s arsenal against COVID-19. Second batch of Sputnik V arrives in Hyderabad, India!, the vaccine developers tweeted. Russia’s Ambassador to India Nikolay Kudashev said in a tweet: given the recent launch of the Russian vaccine in the Indian vaccination campaign, this second delivery has become very timely. Describing Sputnik V as a Russian-Indian vaccine, he told news agency ANI we expect that its production in India will be gradually increased up to 850 million doses per year. There are plans to introduce single-dose vaccine soon in India-Sputnik Lite. Dr. Reddy’s Laboratories, the marketing partner of Sputnik V in India, tweeted that the consignment that arrived today in Hyderabad contains 60,000 doses of the second dose component of the #SputnikV vaccine. Samples from the consignment will be sent for release to the Central Drugs Laboratory. The first consignment consisting of 1.5 lakh doses that arrived in Hyderabad on May 1 was cleared for use on May 13 by the Central Drugs Laboratory in Kasauli. Samples drawn from all consignments would be sent to the central facility in Himachal Pradesh and the vaccine used on getting clearance. Dr. Reddy’s CEO-API and Services Deepak Sapra said this at a media briefing on May 14, following the soft launch of the vaccine by the company as part of a limited pilot. The first dose was administered in Hyderabad. The maximum retail price of the vaccine is ₹995, including a 5% GST. The company said this was the rate at which it would be supplying the vaccine to the government and private sector.

C) DRDO developed drug to be distributed in Delhi.

An anti-coronavirus drug developed by Defence Research and Development Organisation (DRDO) will be launched tomorrow, with Union Defence Minister Rajnath Singh distributing around 10,000 doses to some hospitals in Delhi. The drug, called 2-deoxy-D-glucose or 2-DG, was developed by a DRDO lab in collaboration with Hyderabad-based pharma giant, Dr Reddy’s Laboratories. Drugs Controller General of India (DCGI), the country’s top drug regulator, has approved the medicine for emergency use. The drug had shown promising results in its phase 2 and phase 3 clinical trials. It was also found to be effective in cutting short the hospital stays of patients and reducing supplemental oxygen dependence.

D) Already booked appointments for Covishielf second dose will remain valid.

The Union health ministry on Sunday clarified that already booked online appointments for second dose of Covishield vaccine will remain valid and the same will not be cancelled on Co-WIN platform. It, however, said requisite changes have now been done in the Co-WIN digital portal, as a result of which further online or on-site appointments will not be possible if the period after first dose date for a beneficiary is less than 84 days. The Centre had on May 13 extended the gap between the first and second doses of Covishield vaccine to 12-16 weeks based on the recommendations by the COVID Working Group chaired by N.K. Arora. The Government of India has communicated this change to states and UTs. The Co-WIN digital portal has also been reconfigured to reflect this extension of interval for two doses of Covishield, manufactured by Serum Institute of India (SII) to 12-16 weeks, the Ministry said. However, there have been reports in a section of the media suggesting that people who had pre-booked their appointments for the second dose in less than 84 days on Co-WIN are being turned back from vaccination centres without getting the second dose of Covishield, it said. Additionally, already booked online appointments for second dose of Covishield will remain valid and are not being cancelled by Co-WIN. Further, the beneficiaries are advised to reschedule their appointments for a later date beyond the 84th day from the date of first dose of vaccination, the Ministry added.

E) ‘Entire globe is a unit’ government says while justifying vaccines export.

The entire globe is a unit during a pandemic. This is how the government justified in the Supreme Court its decision to export vaccines amidst a surging second wave of Covid-19. A page in the Ministry of External Affairs’ website shows that between January and April 2021, India exported 663.698 lakh Made in India Covid-19 vaccine supplies to over 90 countries and UN health workers and peacekeepers. Once an epidemic takes (the) form of a pandemic, its management has to be done keeping the entire globe as (a) unit, a March 11 affidavit of the Health Ministry informed the apex court. The document sheds further light into the much criticised-move to export COVID-19 vaccines. In fact, according to the affidavit, the government reasoned it was not possible to take a country or States-specific approach. The government envisaged vaccine export as part of a global action to vaccination. The Centre reasoned that it was necessary to protect the high-risk population in other countries to break the chain of transmission and minimise chances of import of COVID-19 cases to India. India is not immune to the pandemic till the world at large has contained the disease, the Centre argued. It said the export was limited and done giving highest priority to domestic needs. But this is not all. The government, in the affidavit, gives another dimension to why it exported vaccines and opted for staggered immunisation. It said both were done to avoid disproportionality between the production of COVID-19 vaccines and the country’s available health infrastructure and manpower. Simultaneous vaccination without priority classification would have led to commotion, the Centre told the Supreme Court. The affidavit highlights the need for adequate manpower and sufficient infrastructure to cope with the immunisation drive. The document indicates that the available health infrastructure and manpower may not match up. To illustrate, having received one crore vaccine doses for a particular State or city, the vaccine drive would need sufficient number of medical staff who can administer the vaccines and infrastructure like hospitals, primary health centres, etc. It is needless to mention that manufacturing of vaccine would not be proportionate to the available manpower and infrastructure facilities in the country, the government justified.

F) Rahul Gandhi tweets ‘offending’ poster, dares Delhi Police to ‘Arrest me too’

Challenging the arrests of 24 persons by the Delhi police for posters that surfaced across the city questioning Prime Minister Narendra Modi’s decision to donate vaccines to neighbouring countries, former Congress president Rahul Gandhi posted the ‘offending’ poster on his Twitter account daring the police to arrest him. All the posters uniformly pose the question: Modiji humare bachon ka vaccine videsh kyon bhej diya? (Why did you send vaccines meant for our children abroad?). Mr Gandhi said, Arrest me too. Nearly six crore doses of vaccines have been sent out to various countries. So far, a little over four crore persons have got both doses in India and 14 crore people have got a single dose as per the latest numbers on the centralised vaccination website CoWin. The persons were arrested under a rarely used law of Prevention of Defacement of Property Act and most of them have been released on bail. Celebrate, India is a free country. There is freedom of speech, except, when you ask a question of the Honourable Prime Minister, senior Congress leader and former Home Minister P. Chidambaram wrote on Twitter. He said the poster asks a simple question and before PM Modi could answer the Delhi police had answered with arrest. Congress Chief Whip in the Rajya Sabha Jairam Ramesh pasted the poster next to his nameplate at his official residence opposite the Lodhi Garden. He said this smacks of a lawless state gone amuck.

G) Plea in SC to use PM-Cares fund for vaccines, oxygen plants.

A plea was filed in the Supreme Court seeking a direction to utilise the PM-CARES fund for immediate procurement of vaccines and establishment of oxygen plants, generators and their installation in 738 district hospitals across the country. The petition, filed by advocate Viplav Sharma, said the government needs to loosen its PM-CARES purse strings and use the money to help common people urgently access medical care and oxygen. These government hospitals are easily accessible at no cost to common people of every district in the country who are desperately seeking medical oxygen as basic life-saving support, the petition said. The petition also asked for a stay on an April 24 notification of the Centre granting exemption on import duty of medical equipment relating to oxygen generation owing to its acute and rising demand. The petition said the exemption has been wrongfully capped for only three months. People may need this equipment for more than that period. Three months’ cap on the exemption period is too short a period from the standpoint of logistics involved in importing this highly sophisticated medical equipment in India by over 300 hospitals, it said. The plea asked the court to issue directions to States and Union Territories to ensure that private and charitable hospitals have actually procured, installed and commissioned medical plants or equipment with essential backup for medical oxygen for COVID-19 patients. The petition said States and UTs should ensure the setting up of electric and other kinds of crematoriums in cities and improve the existing ones. It also sought the preparation of a ‘National Plan’ in consultation with Chief Secretaries and States.

INTERNATIONAL NEWS 

A) Kurz expects to be charged but cleared in perjury case.

Austrian Chancellor Sebastian Kurz expects to be charged but eventually cleared in an investigation into whether he gave false testimony to a parliamentary commission, he said on Sunday, ruling out the idea of resigning if indicted. The investigation by anticorruption prosecutors, made public, last week poses a stiff political challenge for the conservative Mr. Kurz, 34, who governs in coalition with the Greens. Mr. Kurz has painted himself as the victim of opposition parties trying to trap him into saying something that could be construed as perjury before the commission, which is looking into possible corruption under his previous coalition with the far-right Freedom Party (FPO) which collapsed in 2019. After every word of mine on 58 pages (of testimony) is put on the scale, I certainly expect a criminal complaint, that’s right, he told the Krone newspaper in an interview, adding he had not yet been questioned by prosecutors. But he said he was confident he would be exonerated in the case, which centres on whether he answered truthfully when asked about appointments to state holding company OBAG. He have spoken to numerous lawyers and several university professors. The tenor was always the same: no one can imagine that there will be a conviction here, he told the paper. In a separate interview with the Oesterreich newspaper, he rejected the idea of stepping down if indicted.

B) As U.K. prepares to reopen, B.l.617.2 strain sparks worry.

As England, Scotland and Wales prepares to unlock parts of their economy on Monday, the future roadmap for reopening has been put in doubt over the more transmissible B.1.617.2 strain, or the Indian strain. According to government data, the case numbers of the Indian variant have risen from 520 to 1,313 this week. However, Heath Secretary Matt Hancock expressed confidence that existing vaccines are effective against the new variant. Mr. Hancock said the government had a high degree of confidence that vaccines would stand up to the BI.617.2 variant, following new early data from Oxford University. That means that we can stay on course with our strategy of using the vaccine to deal with the pandemic, he said. The British government has come under criticism from opposition politicians over its decision to put Pakistan and Bangladesh on its red list before India. Mr. Hancock rebuffed the suggestion the decision was influenced by a planned trip by Prime Minister Boris Johnson in April to assist in post-Brexit trade talks. They take these decisions based on the evidence, he said over the visit which was eventually scrapped because of surging COVID-19 cases in India. Indoor hospitality and indoor entertainment such as cinemas, museums and sports venues are to open their doors in most parts of the U.K. for the first time in months on Monday. People and families will also be able to meet with some restrictions in private houses under the new measures. Mr. Hancock said the reopening could go ahead because of the country’s successful vaccination campaign and close monitoring of cases. However, he sounded a note of caution over plans to completely lift restrictions on June 21. We’re in a race between the vaccination programme. And the virus, and this new variant has given the virus some extra legs in that race, but we have a high degree of confidence that the vaccine will overcome, he said. The government’s former chief scientific Adviser said the U.K. now found itself in a perilous movement.  Britain is one of the countries the worst hit with over 127,000 deaths.

Social Science 10th Previous Year Question Paper 2017 (CBSE)

Social Science

SET-I

Section – A

Q. 1. Name the writer of the book ‘Hind Swaraj’. 

Answer: The book ‘Hind Swaraj’ was written by Mahatma Gandhi.

 

Q. 2. Name the river-related to National Waterways No. 2. 

Answer:National Waterways No. 2 is related to the river ‘Brahmaputra’.

 

Q. 3. Explain any one difference between a pressure group and a political party. 

Answer:

Pressure Group Political Party
They have a specific interest and work for the collective interest of their members Eg. Railway employee association, Teacher’s association. They do not directly control or share political powers. They have a broad programme that covers many aspects of national interest.
Their membership is limited. The membership of political parties is broad.

 

Q. 4. Explain the meaning of democracy. 

Answer:Democracy is a form of government in which the supreme power is vested in the people and the representatives of the people are elected by the voters on the basis of adult franchise.

 

Q. 5. Name any one political party of India which grew out of a movement. 

Answer:The political party of India that grew out of movement is ‘Asom Gana Parishad.’

 

Q. 6. How does the use of money make it easier to exchange things ? Give an example. 

Answer:Money makes exchanging things easier as:

  • It is in the form of authorised paper currency which gives the guarantee of the mentioned price to the owner.
  • It has general acceptability.
  • Its price remains constant compared to other commodities.
  • It can be stored easily and doesn’t need much space.

 

Q. 7. Give an example of a violation of the consumer’s right to choose. 

 

Q. 8. How is the maximum retail price printed on packets beneficial for you? 

Answer:Through printed Maximum Retail Price (MRP) on the packets or products, consumers get to know that they do not have to pay prices more than the printed price. Then, the shopkeeper cannot cheat the consumers by overcharging them.

 

Q. 9. Describe any three economic hardships faced by Europe in the 1830s. 

OR

Describe any three problems faced by the French in the sphere of education in Vietnam.

Answer:Economic hardships faced by Europe in 1830s were:

  • Increase in population.
  • Unemployment, migration and price rise.
  • Stiff competition in the market.
  • Bad condition of peasants. (Any three)

OR

The French faced the following problems in the sphere of education in Vietnam:

(i) After receiving western education, the Vietnamese might question colonial domination as it was done by Indians in India under British rule. They would become aware of the western democracies and would put forward various demands before the French government,

(ii) Educated Vietnamese might demand various white-collar jobs or better-paid jobs such as the jobs of teachers, policemen, etc. which were being done by the French citizens called colons living in Vietnam. Thus, there was opposition from the French citizens to give education to the Vietnamese.

(iii) The elites in Vietnam were under the powerful influence of the Chinese culture. It was necessary for the French to counter this influence too.

 

Q. 10. Why did Gandhiji decide to withdraw the ‘Non-Cooperation Movement’ in February 1922? Explain any three reasons. 

Answer:Causes of withdrawal of Non-Cooperation Movement are as follows:

  • Some activists of the Non-Cooperation Movement set a police station on fire at Chauri-Chaura (Gorakhpur), Uttar Pradesh in which 21 policemen were burnt alive.
  • Gandhiji felt that people of India were not ready for a nation-wide movement of mass struggle and felt that he should withdraw the movement.
  • Moreover, many members of the Indian National Congress felt that the Non-Cooperation Movement was tiresome and unnecessary since they wanted to contest the election.

 

Q. 11. Evaluate the role of business classes in the ‘Civil Disobedience Movement.’ 

Answer: The role of business classes in the Civil Disobedience Movement is as follows:

  • Keen on expanding their business, the business classes supported Civil Disobedience Movement and Protested against colonial policies that restricted business activities. They wanted protection against the import of foreign goods and a rupee sterling foreign exchange ratio that would discourage imports.
  • To organise business classes against colonial policies, they formed the Indian Industrial and Commercial Congress in 1920 and the Federation of the Indian Chamber of Commerce and Industries (FICCI) in 1927.
  • The business community interpreted Swaraj in their own way. They came to see Swaraj at the time when the colonial restriction on business would no longer exist and the trade industry would flourish without constraint.

 

Q. 12. Describe any three characteristics of Durg- Bastar-Chandrapur Iron ore belt in India. 

Answer:The characteristics of Durg-Bastar- Chardrapur Iron-ore belt in India are as follows:

  • The Durg-Bastar-Chandrapur belt of high-grade hematite iron ore. This high-quality iron ore is suitable for steel-making.
  • The steel made from this ore is used to produce automobiles, railway equipment and in the defence sector.
  • Half of the iron ore is exported to Japan and South Korea from the Vishakhapatnam port as building a new steel plant is a very costly affair.

 

Q. 13. Analyse the role of the manufacturing sector in the economic development of India. 

Answer:Contribution of the Manufacturing sector to the national economy:

  • Creation of alternative employment: Manufacturing reduces dependence on agriculture by providing alternative employment opportunities in the factory.
  • Better standard of living: The industrial labourers and other employees get higher wages and enjoy a higher standard of living compared to landless agricultural labourers.
  • Support to agricultural production: Increase in use of fertilizers, pesticides, plastics, electricity and diesel in agriculture has been possible due to the growth and competitiveness of the manufacturing industries.
  • In the present day world of territorial specialization, our industry needs to be more efficient and competitive Our goods must be at par with those of other countries in the international market This will fetch foreign exchange and increase national wealth. (Any three)

 

Q. 14. Examine with example the role of means of transport and communication in making our life prosperous and comfortable. 

Answer:Transportation and communication have made our life prosperous and comfortable in the following ways:

(i) Because of transport, raw materials reach the factory and finished products reach to consumers. The pace of development of a country depends upon the production of goods and services as well as their movement over time. Therefore, efficient means of transport are a prerequisite for fast development.

(ii) Apart from transport, the ease and mode of communications, like mobiles, internet, and Wi-Fi makes a seamless flow of information possible.

(iii) Today, India is well-linked with the rest of the world despite its vast size, diversity and linguistic and socio¬cultural plurality. Railways, airways, waterways, newspaper, radio, television, cinema and internet etc., have been contributing to its socio-economic progress in many ways. The trade from local to international level has added to the vitality of its economy. It has enriched our life and has substantially added to growing amenities and facilities for the comforts of life.

 

Q. 15. Analyse the role of popular struggles in the development of democracy. 

Answer:Democracy evolves through popular struggles. It is possible that some significant decisions may take place through consensus and may not involve and conflict at all. But that would be an exception. Defining moments of democracy usually involve conflict between those groups who have exercised power and those who aspire for a share in power. These moments come when the country is going through a transition to democracy, expansion of democracy or deepening of democracy.

(i) Democratic conflict is resolved through mass mobilization. Sometimes it is possible that the conflict is resolved by using the existing institutions like the parliament or the judiciary. But when there is a deep dispute, very often these institutions themselves get involved in the dispute. The resolution has to come from outside, from the people.

(ii) These contacts and mobilizations are based on new political organisations. True; there is an element of spontaneity in all such historic moments. But the spontaneous public participation becomes effective with the help of organized politics. There can be many agencies of organized politics. These include political parties, pressure groups and movement groups.

 

Q. 16. How do pressure groups and movements strengthen democracy? Explain. 

Answer:Pressure groups and movements exert influence on politics in the following ways:

(i) Information campaigns, organising meetings, file petitions: Pressure groups carry out information campaigns, organise meetings and file petitions to attract public attention and gain support for their activities. They influence the media to attract more attention to their issues.

(ii) Protest activities: Pressure groups organise protest activities like strikes, demonstrations or disrupting government’s programmes. Such tactics are employed by worker’s organisations, employees’ associations and most of the movement groups in order to force the government to take note of their demand.

(iii) Lobbying: Business groups employ professional lobbyists or sponsor expensive advertisements. They participate in official bodies and committees that offer advice to the government.

(iv) Take a political stand on major issues: Pressure groups and movements do not directly take part in party politics but try to exert influence on political parties by taking a political stand on different issues. They have their own political ideology and political position on major issues.

Thus, the pressure groups and the movements exert influence on politics in different ways in a democracy. (Any three)

 

Q. 17. On the basis of which values will it be a fair expectation that democracy should produce a harmonious social life? Explain. 

Answer:The basic values of democracy which provide fair expectation that democracy will produce harmonious social life are:

(a) Social Equality, (b) Freedom, (c) Justice, (d) Economic justification, (e) Political freedom, civil and fundamental rights.

Moreover, the following can be understood.

  • Democracy is a form of government in which the supreme power is vested in the people and the representatives of the people are elected by the voters on the basis of adult franchise.
  • It promotes equality among citizens.
  • It looks after the interest of the people.
  • It allows accommodation of social diversity.

 

Q. 18. Explain any three loan activities of banks in India. 

Answer: Activities of banks in India who are involved in providing loan:

  • Banks provide loans for various economic activities.
  • Banks intermediate between those who have surplus funds and those who are in need of these funds.
  • Banks offer very less interest on deposits than what they demand on loans.

 

Q. 19. How do Multinational Corporations (MNCs) interlink production across countries? Explain with examples. 

Answer:Multinational Corporations (MNCs) interlink their production across countries in various ways:

(i) A multinational corporation (MNC) is usually a large company that owns and controls the production in more than one nation. MNCs set up offices and factories for production in regions where they can easily get cheap labour and other resources. This is done to minimise the cost of production end to maximise the profit.

(ii) The MNCs not only sell its finished products globally, but more importantly, the goods and services are produced globally.

(iii) The production process is divided into small parts and spread out across the globe.

(iv) The most common route for MNC investments is to buy local companies and then to expand production.

For example: Cargill Foods, a very large American MNC had bought over an Indian company Parakh foods which had their large marketing network in various parts of India and also has a good reputation. With this advantage, Cargill is now the largest producer of edible oil in India.

(v) Also, MNCs control production by placing orders around the world with a large number of small producers of items, like garments, footwears, sports items, etc. Then MNC sells these products under its brand name.

(vi) As a result, production of MNCs in widely dispersed location is getting interlinked.

 

Q. 20. Analyse the importance of the tree-tier judicial machinery under Consumer Protection Act (COPRA), 1986 for redressal of consumer disputes. 

 

Q. 21. “The first clear expression of nationalism came with the ‘French Revolution’ in 1789.” Examine the statement. 

OR

Examine the reasons that forced America to withdraw from the Vietnam war.

Answer:It is true that “the first clear expression of nationalism came with the ‘French revolution’ in 1789″.

(i) Till 1789, France was a full-fledged territorial state under the rule of an absolute monarch.

The political and constitutional changes that came during the rise of the French Revolution, led to the transfer of sovereignty from monarchy to the body of French citizen.

(ii) The revolution proclaimed that it was the people who would hence forth constitute the nation and shape its destiny.

From the beginning, the French revolutionaries introduced various measures and practices that could create a sense of collective identity, and a feeling of nationalism among the French people. The community was enjoying equal rights under the constitution.

(iii) A new French flag, the tri-colour, was chosen to replace the former royal standard. The Estates General was deleted by the body of active citizens and renamed as National Assembly. New hymns were composed, oaths were taken and martyrs were commemorated, all in the name of nation.

(iv) Regional dialects were discouraged and French, as it was spoken and written in Paris, became the common language of the nation.

The revolutionaries further declared that they would help other people of Europe to become free nations. When the news of the events of France reached different cities of Europe, students and other members of educated middle classes began setting up Jacobin Clubs. Their activities and campaigns prepared the way for the French armies which moved into Holland, Belgium, Switzerland and much of Italy in 1790s.

Thus, with the outbreak of revolutionary wars, nationalism spread in the entire Europe.

OR

Reasons of withdrawal of armed forces from the Vietnam War (The USA War in Vietnam: 1967-January 1974) are as follows:

(i) The phase of struggle with US was brutal as it widely used chemical weapons-Napalm, Agent orange and phosphorus bombs. Besides soldiers, a large number of civilians died in this war.

(ii) The US media and films played a major role in both supporting as well as criticising the war.

(iii) Hollywood made films in support of the war. Such as, John Wayne’s Green Berets (1968).

Writers such as Mary Me Carthy and actors like Jane Fonda even visited North Vietnam and praised their heroic defence of the country.

(iv) The prolongation of the war created strong reactions, even within the U.S. It was clear that the US had failed to achieve its main objective i.e, the Vietnamese resistance had not been crushed, the support of the Vietnamese people for the US action had not been won.

(v) The US war in Vietnam was the most unpopular in history. It was condemned by people all over the world including the United States.

Vietnam soon emerged as a united country. The defeat of the greatest military power in the world by the people of a very small country in Asia was an event of great significance in the history of the contemporary world.

 

Q. 22. How did the Colonial Government repress the ‘Civil Disobedience Movement Explain? 

Answer: The ‘Civil Disobedience Movement’ boycotted foreign cloth and picketed liquor shops. Peasants refused to pay revenue and taxes, village officials resigned. The Colonial Government repressed the members participating in movement.

  • In many places, forest people violated forest laws by going into reserved forests to collect wood and graze cattle. Worried by the developments, the colonial government began arresting the Congress leader one by one. This lead to violent clashes in many places.
  • Abdul Gaffar Khan, a devout disciple was arrested in April 1930. Many people were killed who protested it.
  • When Mahatma Gandhi was arrested, industrial workers of Sholapur attacked police posts, municipal buildings, law courts and railway stations.
  • British government was worried and frightened by this development of movement and it followed a policy of brutal repression.
  • Peaceful Satyagrahis were attacked, women and children were beaten and about one lac people were arrested.

 

Q. 23. Why is it necessary to conserve mineral resources? Explain any four ways to conserve mineral resources. 

Answer:Minerals require millions of year to form. These are non-renewable resources and their stock is limited. Continuous extraction of minerals raises the cost of extraction as they have to be dug from greater depths. Minerals may also be low in quality.

  • A concerted effort has to be made in order to use our mineral resources in a planned and sustainable manner.
  • Use of substitutes in order to save minerals should be encouraged.
  • Improved technologies need to be constantly evolved to allow the use of low-grade ores at low costs.
  • Recycling of minerals using scrap metals and other substitutes are some steps in conserving our mineral resources for the future.

 

Q. 24. Analyse the role of chemical industries in the Indian economy. 

Answer:The chemical industry in India is diversifying and growing fast. It contributes approximately percent to 3% of GDP. It is the third-largest in Asia and occupies twelfth plate in the world in terms of its size. It comprises both large and small scale manufacturing units. Rapid growth has been recorded in both inorganic and organic sectors. Inorganic chemicals include sulphuric acid, nitric acid, alkalies, soda ash and caustic soda. These industries are widely spread over the country.

Organic chemicals include petrochemicals, which are used for manufacturing of synthetic fibres, synthetic rubber, plastics, dye-stuffs, drugs and pharmaceuticals. Organic chemical plants are located near oil refineries and petrochemical plants.

The chemical industry has its own largest consumers. Basic chemicals undergo processing for the further production of other chemicals that are used for industrial application, agriculture or directly for consumer markets.

 

Q. 25. Describe any five characteristics of democracy. 

Answer:The main characteristics of democracy vary in a wide selection of ways.

  • Democracy is a form of government in which the rulers are elected by the people.
  • In a democracy, the final decision of making power must rest with those representatives, elected by people or citizen.
  • Democracy must be based on a free and fair election, where those who are currently in power have a fair chance of losing.
  • In a democracy, each adult citizen must have one vote and each vote must have one value.
  • A democratic government rules within limits set by constitutional law and citizens’ right.

 

Q. 26. “It is very difficult to reform politics through legal ways.” Evaluate the statement. 

Answer:We agree that it is very difficult to reform politics through legal ways. It is very tempting to think of legal ways of reforming politics, to think of new laws to ban undesirable things, but this temptation needs to be resisted. No doubt, the law has an important role to play in political reform. Carefully devised changes in law can help discourage wrong political practices and encourage good ones. But legal constitutional changes by themselves cannot overcome challenges to democracy. This is like the rule of cricket. A change in rules for LBW decisions helped to reduce negative batting tactics. But the improvement is possible mainly by combined efforts by the players, coaches and administrators. Similarly, democratic reforms are to be carried out mainly by political activists, parties, movements and politically conscious citizens.

 

Q. 27. Analyse any five positive effects of globalisation on the Indian economy. 

Answer:The visible impacts of globalisation on Indian economy can be described in the following ways:

(i) There is a wide choice of goods and services in the market.

For e.g., The latest models of digital cameras, mobile phones and television made by the leading manufacturers of the world are available in the markets. These products are affordable as well as within reach of the people.

(ii) Several improvements in the transportation technology has made much faster delivery of goods across long distances possible and that too on lower rates.

(iii) The improvement in information and telecommunication technology is even more remarkable. The invention and use of the computer, internet, mobile phone, fax, etc., have made contact with each other around the world quite easy.

(iv) New jobs have been created in industries where MNCs have invested such as electronics, fast foods, cell phones etc.

(v) Some Indian companies have become multinational by themselves due to globalisation, such as Tata Motors (automobiles), Ranbaxy (Medicines), Infosys (Computer and Information Technology) and L & T (construction).

 

Q. 28. What is liberalisation? Describe any four effects of liberalisation on the Indian economy. 

Answer:The liberalisation of the economy means to free the trade from direct or physical controls imposed by the government.

The four effects of liberalisation on the Indian economy are:

  • Competition would improve the performance of producers within the country.
  • Barriers to foreign trade and foreign investment were removed to a large extent. This meant that goods could be imported and exported easily.
  • Foreign companies could set up factories and offices to boost up production.
  • It allows making decisions freely.
  • The competition would improve the performance of producers within the country since they have to improve their quality.

 

Q. 29. Three features A, B and C are marked on the given political outline map of India. Identify these features with the help of the following information and write their correct names on the lines marked on the map: 

A. The city is associated with the Jallianwala Bagh incident.

B. The place where the Indian National Congress session (1927) was held.

C. The place where Gandhiji violated them salt Law.

CBSE Previous Year Question Papers Class 10 Social Science 2017 Outside Delhi Term 2 Q29

Answer:

CBSE Previous Year Question Papers Class 10 Social Science 2017 Outside Delhi Term 2 Q29.1

 

Q. 30. On the given political outline map of India locate and label the following features with appropriate symbols: 

A. Narora – Nuclear Power Plant.

B. Tuticorin – Major Sea Port.

C. Bhilai – Iron and Steel Plant.

CBSE Previous Year Question Papers Class 10 Social Science 2017 Outside Delhi Term 2 Q30

Answer:

CBSE Previous Year Question Papers Class 10 Social Science 2017 Outside Delhi Term 2 Q30.1

 

SET – II

Q. 1.What is the meaning of ‘Begar’? 

Answer: Begar was a system of forced labour in parts of India. In times, when India was a colony, peasants had to do ‘begar’ and work at landlord’s farms without any payments.

 

Q. 2.Name the best variety of iron-ore found in India. 

Answer:‘Magnetite’ is the finest iron ore with a very high content of iron up to 70%.

 

Q. 3.Why is there overwhelming support to democracy all over the world? Explain one reason. 

Answer:Today, over a hundred countries of the world claim and practice democratic politics, because it provides equality among citizens and improve the quality of decision making.

 

Q. 13.Why are we not able to perform to our full potential in the production of iron and steel in India? Explain any three reasons. 

Answer:Today with 32.8 million tonnes of steel production, India ranks 9th among the world crude steel producers. It is the largest producer of sponge iron. In spite of the large production of steel, per capita consumption per annum is only 32 Kg. India’s export of steel is 2.25% of the global steel trade. Though India has many advantages for the development of this industry, which includes low cost of iron ore, high-grade raw material in proximity, cheap labour and vast growth potential in the home market. Yet we are not able to perform to our full potential largely due to:

  • High cost and limited availability of cooking coal.
  • Lower productivity of labour.
  • Irregular supply of energy.
  • Poor infrastructure.

 

Q. 14.“Tourism industry in India has grown substantially over the last three decades.” Support the statement. 

Answer:The main reasons for the growth of tourism industry in India over the last three decades are:

  • Over 2.6 million tourists visit India every year and this number is increasing every year. It contributes to nearly 21,828 crore rupees as foreign exchange. More than 15 million people are directly engaged in the tourism industry.
  • It has pleasing weather conditions throughout the year for the tourists in comparison to western countries.
  • Foreign tourists visit India for heritage, medical, economic, culture, business, adventure tourism, etc.
  • It had made access easier in the North-Eastern states and interior parts of Jammu and Kashmir, Himachal Pradesh and Uttranchal.
  • It has become easier nowadays to transport valuable goods, life-saving drugs, perishable commodities, mail, etc., to distant places in a short time.

Thus, it plays an important role in popularising world-class monuments around the world which enhances Indian tourism and contributes to its economy.

 

Q. 15.How is social diversity accommodated in a democracy? Explain with examples. 

Answer:It is a fair expectation that democracy should produce a harmonious social life. Democracy must fulfil two conditions in order to achieve social harmony even though there is social diversity.

(i) It is necessary to understand that democracy is not simply ruled by majority opinion. The majority always needs to work with the minority so that, the possibility of tensions, becoming explosive or violent reduces. The majority and minority opinions are not permanent.

(ii) It is also necessary that the rule of the majority does not become the rule of the majority community in terms of religion or race or linguistic group. Rule of the majority means that in case of every decision or every election, different persons and groups may and should form a majority.

 

Q. 22.How was the sense of collective belonging developed during the freedom movement? Explain. 

Answer:Nationalism spread in India when people began to believe that they were all part of the same nation when they discovered some unity that bound them together. This sense of collective belonging came partly through the experience of united struggle. It was in the 20th century, with the growth of nationalism that the identity of India came to be visually associated with the image of Bharat Mata, created by Bankim Chandra Chattopadhyay.

In 1870, he wrote “Vande Matram” as a hymn to the motherland.

Ideas of nationalism also developed through a movement to revive Indian folkore.

Nationalist began recording folk tales sung by bards and they toured villages to gather folk song and legends. As the national movement developed, nationalist leaders became mdre and more aware of such icons and symbols in unifying people and inspiring in them a feeling of nationalism. In 1921, Gandhiji had designed the swaraj flag.

It was a tricolour (red, green and white) and had a spinning wheel in the centre. Carrying this flag, holding it aloft, during marches became a symbol of defiance.

Another means of creating a feeling of nationalism was through a reinterpretation of history. Indians began looking into the past to discover India’s great achievements. They wrote about the glorious developments in ancient times when art and architecture, science and mathematics, religion and culture, law and philosophy had flourished and decline in these areas began when India was colonised.

 

Q. 23.“The advancement of international trade of a country is an index of its economic development.” Justify the statement. 

Answer:“Advancement of international trade of a country is an index of economic development”.

The statement can be justified through the following point:

  • As no country is self-sufficient in all resources, it cannot survive without international trade.
  • If the balance of international trade is favourable in a country, it will be able to earn more foreign exchange.
  • International trade encourages a country to develop secondary and tertiary sectors for exporting those goods which can fetch more foreign exchange.
  • A country’s economic development and prosperity can be gauged by the health of its international trade.
  • A country can earn a large amount of foreign exchange through international trade. India exports approximately 7500 commodities to about 190 countries and imports about 6000 commodities from 140 countries. India exported commodities worth the US $ 318.2 billion in 2014. And India’s share in export is increasing every year.

 

Q. 26.Suggest any five effective measures to reform political parties. 

Answer:Following are the five effective measures to reform political parties:

  • A law should be made to regulate the internal affairs of political parties.
  • It should be made compulsory for political parties to maintain a register of its members.
  • It should be made mandatory for political parties to give a minimum number of tickets (about 1/3rd) to its woman candidates.
  • There should be a quota for the woman in the decision making bodies of the parties.
  • The government should give parties money to support their election expenses in kind, for example, petrol, paper, phone bill, etc.

SET- III

Q. 1.What is meant by Satyagraha? 

Answer:Satyagraha is a non-violent method of mass agitation against the oppressor. The idea of Satyagraha emphasised the power of truth.

 

Q. 2. Where do minerals occur in igneous and metamorphic rocks? 

Answer:In igneous and metamorphic rocks, minerals are formed due to effects of heat and pressure when magma or lava cools. Minerals in these rocks are present in cracks and joints. There are various type of minerals found in these rocks like tin, copper, zinc, lead, diamond etc.

 

Q. 3. Explain the meaning of transparency in democracy. 

Answer:In democracy, every citizen has the right and the means to examine the process of decision making. This is known as transparency.

 

Q. 13. “Minerals are unevenly distributed in India.” Support the statement with examples. 

Answer:“Minerals are unevenly distributed in India.” Following examples are:

  • Minerals in Deccan: The peninsular rocks contain most of the reserves of coal, metallic mineral, mica, and many other non-metallic minerals.
  • Minerals in western and eastern regions of India: Sedimentary rocks of the western and eastern part of India i.e., Gujarat and Assam have most of the petroleum deposits.
  • Minerals in Rajasthan: Rajasthan with the rock systems of the peninsular has reserves of many non-ferrous minerals.
  • Minerals in North India: The vast alluvial plains of North India are almost devoid of economic minerals. These variations exist largely because of the differences in the geological structure, processes and time involved in the formation of minerals.

 

Q. 14. Evaluate any three features of ‘Golden Quadrilateral’ Super Highways. 

Answer:The three features of ‘Golden Quadrilateral’ Super Highways are:

  • It is a broad network of six lanes of superhighways including North-South Corridor and East-West Corridor. It is a major road development project that connects Delhi- Kolkata-Chennai-Mumbai.
  • Delhi National Highway Authority of India (NHAI) is implementing this gigantic project. With the Quadrilateral road network, the movement of trade has become easy from one comer to other comers in India.
  • This road network is connected to 10 major ports, namely Kandla, Nhava-Sheva, Mormugao, Tuticorin, Chennai and Ennore, Vishakhapatnam, Paradeep, Haldia and Mumbai.

 

Q. 15. Analyse the role of opposition political parties in a democracy. 

Answer:Opposition political parties play an important role in democracy. They play both positive and negative roles.

Positive Role:

  • They ensure that political party, in power, does not, abuse or misuse its power.
  • It exposes the weaknesses of the ruling party.
  • It keeps a close vigil on the bills and expenditure of the government.

Negative Role:

  • It targets the government and aims at the lodging and discrediting the government for all the ills and troubles that people face.
  • Through stalling the proceedings of the parliament, dharmas and gheravs, it curtails the progress of the country.
  • Sometimes, opposition party opposes every decision of the ruling party whether it is in the favour of the nation or not.

 

Q. 22. Explain the measures taken by Gandhiji to eliminate the problem of untouchability. 

Answer:The measures taken by Gandhiji to eliminate the problem of untouchability were:

  • He said that swaraj would not come for a hundred years if untouchability was not eliminated.
  • Gandhiji himself cleaned toilets to dignify the work of a sweeper.
  • Gandhiji persuaded the upper caste to change their heart and give up ‘sin of untouchability’.
  • He organised satyagraha to secure their entry into temples, access to public wells, tanks, roads and public schools.
  • He signed the Poona Pact in September 1932 with Dr. B. R. Ambedkar through which some seats were reserved for the depressed classes in the provincial and central legislative council.

 

Q. 23. Examine any five factors affecting the location of industries in India. 

Answer: Maximisation of profit which also implies cost minimisation is the most important goal in their choice of a particular place for the location of industries. Some factors influencing the same are:

  • Proximity to markets: Areas or regions having high purchasing power provide large market, therefore, luxury items producing industries are located in these regions.
  • Availability of raw material: Raw material used by industries should be cheap and easy to transport. Industries based on cheap, bulky and weight losing material (ores) are located close to the sources of raw material. Such as steel, sugar, and cement industries.
  • Availability of labour: Some types of manufacturing requires skilled labour there-fore IT industries 
  • Turn on screen reader support
  • Show side panel
  • are located near urban-educational centres where skilled labours are easily available.
  • Access to sources of energy: Industries which use more power are located near energy supplying sources such as the aluminium industry.
  • Transportation and communication facilities: Speedy and efficient transport facilities reduce the cost of transport. Therefore, industries are attracted to regions which have good transport facilities.
  • Government policy: Governments adopt ‘regional policies’ to promote ‘balanced’ economic development and hence set up industries in backward and tribal areas. (Any five)

 

Q. 26. “No party system is ideal for all countries and in all situations.” Analyse the statement. 

Answer:‘No party system is ideal for all countries and in all situations’.

This statement can be justified as follows:

  • All countries and democracies have different social situations, economic achievements and culture and thus different political situations leading to different party systems.
  • Party system is not anything that a country can choose. It evolves gradually and slowly depending on the situations in the country.
  • Party system depends on the country’s social and regional differences, its history of politics and elections system.
  • India has evolved a multiparty system because its regional and geographical differences are not easily absolved by even two parties.
  • But not all countries have such diversity and so may not need multiparty systems. In such countries, a two-party system may be enough for the representation of various communities. Thus, no party system is ideal for all countries and in all situations.

Latest Current Affairs 16 May 2021

CURRENT AFFAIRS
16 May 2021

NATIONAL NEWS:

 

A) Delhi Police arrests 15 people, lodges 17 FIRs over posters critical of Modi’s handling of vaccination drive.

Delhi Police has registered 17 FIRs and arrested 15 people for allegedly pasting posters critical of Prime Minister Narendra Modi in connection with the vaccination drive against Covid-19, officials said on Saturday, PTI reported. The posters reading ‘Modiji humare bachon ki vaccine videsh kyu bhej diya (PM why did you send vaccines of our children to foreign countries?) were pasted in several parts of the city, they said. On Thursday, police received information about the posters, following which senior officers of the districts were alerted. And based on further complaints, as many as 17 FIRs were registered under sections 188 (disobedience to order duly promulgated by public servant) of the Indian Penal Code and other relevant sections including section 3 of the Prevention of Defacement of Property Act across various districts of the Delhi Police, the officials said. A senior police officer said, More FIRs are likely to be registered if further complaints are received in this regard. As of now, investigation is underway to ascertain as to on whose behalf these posters were being put up at various places across the city and accordingly further action will be taken in the matter. Delhi Police comes under the purview of the Central government, and reports to the Ministry of Home Affairs.

B) Plea in SC seeks CBSE, ICSE Class 12 exams cancellation.

A petition has been filed in the Supreme Court to direct the authorities to cancel the Central Board of Secondary Education (CBSE) and the Indian Certificate of Secondary Education (ICSE) exams for Class 12 students. The petition asked both the boards to instead conceive an objective methodology to declare the Class 12 results within a specific time-frame. Issue a writ of mandamus directing the respondents to cancel the examination of Class 12 and devise an objective methodology to declare the result within a specific time-frame, advocate Mamta Sharma, who filed the petition, urged. He said the CBSE and ICSE notifications deferring the exams to an unspecified date should be quashed. Students cannot be made to suffer uncertainty in the midst of an unprecedented public health crisis posed by the Covid-19 pandemic. There should not be any uncertainty regarding exams crucial to the future academic study of Class 12 students, the plea stated. Last year, the apex court asked the boards to determine and declare Class 12 exam results on the basis of their earlier grading. The petitioner said the same methodology could be used this year too. Both the boards cannot remain mute spectators and opt to wait and watch for the pandemic wave to ebb. Delay would put the future of the students in peril. The boards had cancelled the Class 10 exams. The same should be done for Class 12 students. As far as the innocent students of Class12 are concerned, a step-motherly, arbitrary, inhuman direction have been issued to postpone their final examination for an unspecified duration instead of following the directions propounded and accepted by them last year, the petition said.

C) Govt must explain massive under-reporting of Covid-19 deaths in Gujarat, says Congress.

The Congress on Saturday raised the issue of under-reporting of Covid-19 deaths in some states, especially in Gujarat, and demanded an explanation from both the central and the state governments. Congress leaders P Chidambaram and Shaktisinh Gohil in a joint press conference pointed out that the deaths in Gujarat this year were more than double the number in 2020 and said that the substantial increase can only be attributed to a pandemic. The two Congress leaders cited a news report which showed that Gujarat had issued about 1,23,000 death certificates between March 1 and May 10, as against about 58,000 certificates issued during the same period last year, and said they got these verified after collecting data from 33 districts of the state. The Congress leaders said the sum of the number of the death certificates collected nearly tallies with the numbers published and that comes to 1,23,873 in 2021 against 58,068 last year. However, during the period March 1 to May 10, the government of Gujarat has officially admitted to only 4,218 Covid-related deaths. Chidambaram said the difference between the increase in the number of death certificates (65,805) and the official Covid-related deaths (4,218) must be explained. It cannot be explained as natural annual increase’ or due to other causes, he noted. They have a strong suspicion that the bulk of the increased number of deaths is due to COVID and the state government is suppressing the true number of Covid-related deaths. They have a strong suspicion that the Government of India, in conjunction with some state governments, is suppressing the true numbers of new infections and Covid-related deaths. If our suspicions are true, this is a grave misdeed apart from being a national shame and a national tragedy, he said.

D) Government’s ‘disastrous’ vaccination policy’ will ensure third wave, says Rahul.

Congress leader Rahul Gandhi on Saturday called for a national vaccine strategy and claimed that the government’s disastrous inoculation policy will ensure a devastating third wave in the country. He also accused the Prime Minister of making mother Ganga cry after bodies of suspected coronavirus victims were found floating in the river. The Government of India’s (GOI’s) disastrous vaccine strategy will ensure a devastating third wave. It can’t be repeated enough. India needs a proper vaccine strategy, he said on Twitter. Tagging media reports which had documented that over 2,000 bodies were found in a 1,140 km area along the Ganga, he said, One who used to say ‘Ganga’ has called him has made Mother Ganga cry. Gandhi and the Congress have been attacking the prime minister and his government over its vaccine strategy and handling of the pandemic.

E) Cyclone Tauktae has intensified, moving towards Gujarat: IMD.

Cyclonic storm Tauktae has intensified and is heading towards the coast of Gujarat and the Union Territory of Daman and Diu and Dadra and Nagar Haveli, though it could bring gusty winds and showers to Mumbai, the India Meteorological Department (IMD) said on Saturday. The cyclonic storm is very likely to intensify further into a very severe cyclonic storm by late Saturday night, the IMD said, adding it was very likely to move north-northwestwards and cross Gujarat coast between Porbandar and Naliya around May 18. As it would bring very heavy rainfall in that region, cities like Mumbai would not be affected much, the IMD added. There will be strong winds and heavy rainfall at isolated places on May 17 over north Konkan, including Mumbai, the met department said.

F) Covid Watch: Numbers and Developments.

The number of reported coronavirus cases from India stood at 2,45,77,613 with the death toll at 2,68,139. Bodies of the deceased are buried in the sand near the banks of the Ganga allegedly due to shortage of wood for cremation during the second wave of coronavirus at Shringverpur Ghat in Prayagraj on May 15, 2021. The West Bengal government on Saturday announced a complete lockdown from Sunday till May 30 in order to contain the spread of Covid-19 in the State. We are taking some strict measures to contain the pandemic, starting Sunday 6 am till 6 pm of May 30, Chief Secretary Alapan Bandyopadhyay said. During this period, all government and private offices, shopping complexes, malls, bars, sports complexes, pubs and beauty parlours will remain closed, he said. Movement of private vehicles, taxis, buses, metro rail, suburban trains will also be disallowed during the 15- day lockdown period. Meanwhile, in Tamil Nadu, the vaccination programme for people aged between 18 and 45 will start in two or three days. The State’s Minister for Medical and Family Welfare Ma. Subramanian on Saturday said that Tamil Nadu had already paid ₹46 crore for 15 lakh doses of vaccines, Covishield and Covaxin, of which five lakh doses have arrived. The drive to vaccinate people in the18-45 age group will soon start in one of the locations in Chennai in two to three days, said the Minister after holding a review meeting in Coimbatore along with Food Minister R. Sakkarapani and Forests Minister K. Ramachandran.

INTERNATIONAL NEWS

A) Israeli air strikes destroy Gaza building that housed media outlets, kills several Palestinian children.

An Israeli airstrike destroyed a high-rise building in Gaza City that housed offices of The Associated Press and other media outlets on Friday, the latest step by the military to silence reporting from the territory amid its battle with the militant group Hamas. The strike came nearly an hour after the military ordered people to evacuate the building, which also housed Al-Jazeera, other offices, and residential apartments. The strike brought the entire 12-storey building down, collapsing with a gigantic cloud of dust. There was no immediate explanation for why it was attacked. A fireball and smoke billow up into the air during an Israeli airstrike on Gaza City, in Gaza Strip on May 15, 2021. The strike came hours after another Israeli air raid on a densely populated refugee camp in Gaza City killed at least 10 Palestinians from an extended family, mostly children, in the deadliest single strike of the current conflict. The latest outburst of violence began in Jerusalem and has spread across the region, with Jewish-Arab clashes and rioting in mixed cities of Israel. There were also widespread Palestinian protests Friday in the occupied West Bank, where Israeli forces shot and killed 11 people. The spiralling violence has raised fears of a new Palestinian intifada, or uprising at a time when there have been no peace talks in years. Palestinians on Saturday were marking Nakba (Catastrophe) Day, when they commemorate the estimated 700,000 people who were expelled from or fled their homes in what was now Israel during the 1948 war surrounding its creation. That raised the possibility of even more unrest. U.S. diplomat Hady Amr arrived Friday as part of Washington’s efforts to de-escalate the conflict, and the U.N. Security Council was set to meet Sunday. But Israel turned down an Egyptian proposal for a one-year truce that Hamas rulers had accepted, an Egyptian official said Friday, to discuss the negotiations. Since Monday night, Hamas has fired hundreds of rockets into Israel, which has pounded the Gaza Strip with strikes. In Gaza, at least 139 people have been killed, including 39 children and 22 women; in Israel, eight people have been killed, including the death Saturday of a man killed by a rocket that hit in Ramat Gan, a suburb of Tel Aviv. The strike on the building housing media offices came in the afternoon, after the building’s owner received a call from the Israeli military warning that it would be hit. AP’s staff and others in the building evacuated immediately. Al-Jazeera, the news network funded by Qatar’s government, broadcast the airstrikes live as the building collapsed.

B) Cairn Energy sues Air India in U.S. court to enforce $1.2 billion arbitration award.

Cairn Energy has sued flagship carrier Air India to enforce a $1.2 billion arbitration award that it won in a tax dispute against the country, according to a U.S. District Court filing reviewed by Reuters. The move ratchets up pressure on the Union government to pay the sum of $1.2 billion plus interest and costs that the British firm Cairn was awarded by an arbitration tribunal in December. The body ruled India breached an investment treaty with Britain and said New Delhi was liable to pay. Cairn filed the lawsuit on May 14 in the U.S. District Court for the Southern District of New York, seeking to make Air India liable for the judgment that was awarded to Cairn. The lawsuit argued that the carrier, as a state-owned company, is legally indistinct from the state itself. The nominal distinction between India and Air India is illusory and serves only to aid India in improperly shielding its assets from creditors like [Cairn], the filing said. Air India and the Union government did not immediately respond to requests seeking comment. In February, Cairn filed a separate case in a U.S. court to recognise and confirm the arbitration award, including payments due since 2014 and interest compounded semi-annually. Responding to queries about the case filed in a U.S. Court, a company spokesperson said, Cairn is taking the necessary legal steps to protect shareholders’ interests in the absence of a resolution to the arbitral award. Cairn remains open to continuing constructive dialogue with the Government of India to arrive at a satisfactory outcome to this long-running issue.

Latest Current Affairs 15 May 2021

CURRENT AFFAIRS
15 May 2021

NATIONAL NEWS:

 

A) Petition in Supreme Court asks for vaccine clinical trial data to be made public.

A petition has been filed in the Supreme Court to make public the segregated data of clinical trials of Covid-19 vaccines administered under the Emergency Use Authorisation granted by the Drugs Controller General of India (DCGI). Former member of National Technical Advisory Group on Immunisation Dr. Jacob Puliyel, represented by advocate Prashant Bhushan, asked the court to direct the government, its bodies and the vaccine manufacturers – the Serum Institute of India and Bharat Biotech to transparently reveal clinical trial and vaccination data, including the recording and reporting of adverse events. Further, the petition urged the court to direct the government to not issue any coercive mandates for use of these inadequately tested vaccines. Bhushan argued that courts should reiterate that vaccine mandates are repugnant to the right of humans to autonomy and right to self-determine what may be injected into their bodies. The respondents (Centre, its agencies like DCGI and ICMR and vaccine manufacturers) have maintained opacity about the clinical trial data of the two vaccines administered through emergency authorisation in India. Non-disclosure of this important data violates the basic ethics of clinical research that requires results of clinical research studies to be published and brought to the knowledge of the medical community, participants to the research, and the general population, the petition said. Dr. Puliyel, also represented by advocate Cheryl d’Souza, said the lack of transparency raised concerns over the efficacy and safety of the two vaccines. The World Health Organisation (WHO) has released a strong statement advocating for public disclosure of all clinical trial results. According to the statement, when data is not released it means that doctors, patients and medical regulators cannot make informed decisions about which treatments are best, the petition stated. The petition said transparency in publishing clinical trial data by the Central Drugs Standard Controls Organisation (CDSCO) that granted final approval for the vaccines by various manufactures to enter the immunisation chain flowed from Section 4 of the Right to Information Act, 2005, which required the government to make proactive disclosures of its records through the internet and other means of communications to the general public.

B) Vice President, Lok Sabha Speaker say no to virtual meetings of Parliamentary panels.

The parliamentary panels cannot meet virtually till the necessary rules are amended by both Houses of Parliament, the Rajya Sabha secretariat said in reply to a letter written by Leader of the Opposition Mallikarjun Kharge on Friday. The standing committees of Parliament, which are non-partisan platforms to analyse the functioning of the government, have not met even once since the onset of the second wave of the pandemic. More than a year after the demands were first raised by the panels’ chairpersons, the necessary changes in the rules have not been amended since their physical meetings were held for a few months when it was believed that Covid-19 was on the wane. Last week, in a letter to Rajya Sabha Chairman Venkaiah Naidu, Kharge said that Parliament could not be a mute spectator to the suffering of the people. He urged Naidu to allow online meetings of the panels. Naidu and Lok Sabha Speaker Om Birla last year held a series of meetings where it was argued that the confidentiality clause dictated that only physical meetings of the panels can be held, keeping the proceedings and deliberations secret. It was decided that the issue be referred to the committee on Rules in both the Houses. As physical meetings of the committees were being held regularly, following the guidelines strictly, the matter rested there and the situation had not arisen for considering the matter by the Rules Committees in both the Houses, the letter stated. The Rajya Sabha secretariat assured Kharge that the meetings of the committees would be considered shortly once the situation improves for better. It was informed that the issue of confidentiality can be resolved during the session as any amendment to the rules can be approved by the respective Houses only after the matter is considered by the Rules Committee. Senior Congress leader and chairman of the committee on Home Affairs Anand Sharma who, on Tuesday, wrote to Naidu making the same demand, said: In a democracy, accountability can neither be delayed nor evaded. His colleague and chairman of the panel on Science and Technology Jairam Ramesh wrote on Twitter that nowhere in the world had Parliament run away from its duties like in India. In spite of repeated requests for almost a year, virtual meetings of Standing Committees have been inexplicably disallowed. The PM has all his meetings virtually, but 30 odd MPs cannot, he said.

C) Congress slams Centre as Delhi Police question Youth Congress chief over Covid-19 relief work.

The Congress on Friday asked if providing oxygen cylinders and life-saving drugs to Covid-19 patients is a crime under the Narendra Modi government. This follows the questioning of the Indian Youth Congress (IYC) chief B V Srinivas by the crime branch of the Delhi Police. Today every person has to decide if providing oxygen, helping people to get life-saving drugs like Remdesivir, arranging beds or providing food to ambulance drivers is a crime. It seems to be a crime according to Prime Minister Narendra Modi, said Congress chief spokesperson Randeep Surjewala. Today, Modi ji and [Home Minister] Amit Shah Ji sent Delhi Police to the office of the IYC office to question Srinivas ji. Can there be a more despicable act than this? Is it a sin committed by the Youth Congress and the Congress to provide help that the Modi government has been unable to do? he stated. The government should be ashamed of itself, he said. He asserted that the Youth Congress would continue to provide oxygen and other life-saving drugs to needy patients. They [Delhi Police officials] wanted to know the details of how are we helping people. We answered all their questions, Srinivas said. Using hash tag #IStandWithIYC to express solidarity, former Congress chief Rahul Gandhi tweeted, The one who saves is always greater than the one who destroys. While former Union Minister Jairam Ramesh tweeted to call the Delhi Police’s move as atrocious, party general secretary Priyanka Gandhi Vadra too reacted on Twitter when former Rajya Sabha member Shahid Siddiqui said that a police official had also come to his place to find out how Vadra mananged to help him with Remdesivir injections for his wife. If helping someone in need is now a crime, I will commit it again and again. To my mind it’s a far greater crime to silently watch and do nothing while people die desperately searching for medicine and gasping for air, Vadra said, lending her support to the IYC. At an online press conference, All India Congress Committee in charge of Delhi Shaktisinh Gohil said Delhi Pradesh Congress Committee chief Anil Chaudhary and former MLA Mukesh Sharma also received notice for such questioning. A Delhi Police official claimed that the questioning followed a direction from the Delhi High Court on a writ petition filed by Dr. Deepak Singh about politicians involved in illegal distribution of Covid-19 medicines, etc. 

D) Sputnik V to be priced at ₹995.40 per dose.

Sputnik V has now joined India’s immunisation programme against Covid-19, with the first dose of the Russian vaccine being administered in Hyderabad today. The maximum retail price of the imported vaccine is ₹995.40 per dose. As part of a limited pilot, the soft launch of the vaccine has commenced and the first dose was administered in Hyderabad on May 14, said Dr. Reddy’s Laboratories, the marketing partner in India for the vaccine of the Russian Direct Investment Fund (RDIF). It follows the imported doses of Sputnik V that arrived in the country on May 1, receiving regulatory clearance from the Central Drugs Laboratory, Kasauli, on May 13. Further consignments of Sputnik V are expected over the upcoming months. Subsequently, supply of the vaccine will commence from Indian manufacturing partners.

E) Many districts in Kerala on rain alert.

District administrations in Kerala have scrambled to deal with a potential rainfall-related emergency, with the turbulent weather conditions over the Arabian Sea expected to trigger torrential rains over the next few days. Five Kerala districts Thiruvananthapuram, Kollam, Pathanamthitta, Alappuzha and Ernakulam were on red alert today (Friday), given the likelihood of extremely heavy rainfall, according to a corrected weather update issued by the India Meteorological Department (IMD) at 1 p.m. In a 10 a.m. update, the weather agency had downgraded the red alerts that were sounded in three districts on Friday to orange alerts. As per the latest update, Kottayam, Idukki, Thrissur, Palakkad , Malappuram, Kozhikode and Wayanad are on orange alert today (Friday) and Kannur and Kasaragod on yellow alert.

F) Covid Watch: Numbers and Developments.

The number of reported coronavirus cases from India stood at 2,41,00,326 with the death toll at 2,62,951. Lockdown restrictions in Tamil Nadu are set to become more stringent from tomorrow. Provision and grocery stores and shops selling meat would be allowed to function only from 6 am to 10 am across Tamil Nadu from Saturday. Tea shops will be closed. These shops were earlier allowed to remain open from 6 am to 12 noon. E-registration would be made mandatory from May 17 for inter-district and intra-district movement even for essential purposes, weddings or funerals of close relatives and also for travelling for medical treatment and for elderly care, a Government release said on Friday evening. Shops on pavements, which were earlier allowed to sell vegetables, flowers and fruits till 12 noon, would not be allowed any more. In neighbouring Kerala, Chief Minister Pinarayi Vijayan on Friday extended the state-wide lockdown till May 23. Vijayan said the government is taking steps to mitigate the ill-effects of the lockdown, chiefly loss of livelihood. He announced free food kits for the population and immediate distribution of social welfare pensions. The government would support Anganwadi teachers and Kudumbashree units, he added.

INTERNATIONAL NEWS 

A) Oli reappointed Nepal PM. 

K.P. Sharma Oli, heading a minority government, was sworn in as Nepal’s Prime Minister on Friday, four days after the embattled leader lost a crucial vote of confidence in Parliament. The 69-year-old Chairman of the Communist Party of MarxistLeninist) was reappointed as Prime Minister by President Bidya Devi Bhandari in his capacity as leader of the largest political party in Nepal’s House of Representatives. Mr. Oli will head a minority government as he does not enjoy a majority in Parliament after losing the vote of confidence on Monday. He was reappointed to the post on Thursday night as the Opposition parties failed to secure majority seats in Parliament. Mr. Oli will now have to take a vote of confidence at the House within 30 days, failing which, an attempt to form a government under Article 76 (5) of the Constitution would be initiated by the President. The Ministers of Mr. Oli’s Cabinet were also sworn in during the ceremony. All the Ministers from the old Cabinet have been included in the new Cabinet. Pradeep Gyawali has been reappointed as Foreign Minister while Ram Bahadur Thapa and Bishnu Poudyal were appointed as Ministers for Home and Finance.

 

B) Kabul mosque bombing kills 12 devotees. 

A bomb ripped through a mosque in northern Kabul during Friday prayers, killing 12 worshippers, and wounding 15, Afghan police. No one immediately claimed responsibility for the bombing, the latest in a surge in violence as U.S. and NATO troops have begun their final withdrawal from the country, after 20 years of According to Afghan police spokesman, Ferdaws Faramarz, the bomb exploded as prayers had begun. The mosque’s Imam, Mofti Noman, was among the dead, the spokesman said and ad, ded that the initial police investigation suggests the Imam may have been the target. Taliban spokesman Zabihullah Mujahid denied any insurgent connection to the mosque attack, condemning it and accusing Afghanistan’s intelligence agency of being behind the explosion. Both the Taliban and government routinely blame each other for attacks. The attackers are rarely identified, and the public is seldom informed of the results of investigations into the many attacks in the capital. One worshipper, Muhibullah Sahebzada, said he had just stepped into the building when the explosion went off. Stunned, he heard the sound of screams, including those of children. as smoke filled the mosque. The explosion comes on the second day of a three-day ceasefire announced by the Taliban for Id-ul-Fitr, follows the fasting month of Ramzan. The Afghan government has said it would abide by a truce. So far, many of the attacks in Kabul have been claimed by the Islamic State group’s lcxal affiliate, though the Taliban and government routinely trade blame.

×

Hello!

Click one of our representatives below to chat on WhatsApp or send us an email to info@vidhyarthidarpan.com

×